Акдс сколько раз ставят: Как АКДС влияет на здоровье детей?

Содержание

Если пропущена прививка — 2

Вопрос:

Ребенку сейчас 1,5 года, первая АКДС была сделана вакциной Инфанрикс, вторая в год Пентаксим, третью сейчас надо делать, но ждем когда появится Инфанрикс, можно ли сейчас делать прививку Корь краснуха паротит, пока не сделана третья АКДС?И если не появиться Инфанрикс, можно ли будет делать Пентаксим? у нас в 2 года должна быть ревакцинация от полиомиелита?

Ответ

Отвечает Харит Сусанна Михайловна

Ребенок раннего возраста должен иметь 4 прививки против коклюша и 5 прививок против полиомиелита. У вас нарушен график прививок. Поэтому если нет случаев кори, можете сделать Пентаксим, через 6-9 месяцев ревакцинацию Пентаксима, в промежутке- реакцию Манту и прививку против кори, краснухи, паротита. Если же в вашем регионе есть случаи кори, то прививайтесь сначала против кори, краснухи, паротита, а затем через 1-1,5 месяца продолжите Пентаксим по вышеуказанной схеме. Не забудьте также вакцинацию против пневмококковой инфекции – 2 прививки с интервалом в 2 месяца, а также против вирусного гепатита В.

Вопрос:

Ребенку была поставлена вакцина от полиомиелита (ИПВ) в июле 2014 года. Затем мы прекратили вакцинацию. А теперь продолжим. Вопрос: надо ли начинать с начала или продолжать уже начатую вакцинацию? То есть считать ли ту первую введенную вакцину и еще доставить две или ставить всё же еще три?

Ответ

Отвечает Полибин Роман Владимирович

Если пропущена вторая прививка от полиомиелита, то вакцинацию снова не начинают, а продолжают, несмотря на увеличенный интервал.

Вопрос:

Первую прививку от гепатита В сделали в 7 месяцев. Потом был большой перерыв. Вторую сделали в год и 6 месяцев, а третью — в год и 7 месяцев. Нужно ли делать еще одну прививку, так как между первой и второй был большой перерыв?

Ответ

Отвечает Полибин Роман Владимирович

График, по которому делаются прививки от гепатита желательно соблюдать в точности. Если прививка задерживается более чем на три месяца, вся схема начинается заново, с первой прививки, то есть Вам необходимо будет сделать еще одну прививку в два года. (схема 0-1-6). Однако, в инструкции по применению «ЭНДЖЕРИКС — В» указано: в случае удлинения интервала между первой и второй прививками на 5 мес. и более третью прививку проводят через 1 мес. после второй.

Вопрос:

Дочке прививку АКДС сделали в 3 месяца после этого больше не делали, ей сейчас уже 1,9 месяцев, хотим возобновить прививки. 1 вопрос: первые 3 прививки АКДС делаются, чтобы ребенок не заболел до 2лет или на всю жизнь.2 вопрос: если мы пропустили эти прививки, как теперь привить пожизненно ребенка?

Ответ

Отвечает Полибин Роман Владимирович

Ответ на 1 вопрос: Вакцина АКДС обеспечивает детям иммунитет от коклюша, столбняка и дифтерии более чем на 5 лет, о чём знают многие родители. Однако, куда меньшее количество родителей вдаётся в тонкости иммунологии, не подозревая, что приобретённый впервые иммунитет от коклюша и столбняка в 15–20% случаев пропадает уже через год после вакцинации. Организм перестаёт считать инфекцию реальной угрозой в дальнейшем и постепенно перестаёт вырабатывать антитела. Чтобы это предотвратить, детям следует сделать ещё одну дополнительную прививку, которая даст 100% иммунный ответ на необходимый срок. Многие родители, не зная этого, отказываются от такой скорой повторной вакцинации АКДС, особенно если у малыша наблюдались серьёзные реакции в первый раз. Важно: если ребёнок всё же окажется в 20% детей, потерявших иммунитет после первых уколов АКДС, он будет беззащитен перед тремя самыми опасными инфекционными заболеваниями до 6 лет. Установить это точно без серьёзного иммунологического исследования невозможно, поэтому легче просто сделать лишнюю прививку.

Ответ на 2 вопрос: Если ребенку прививку делают с нарушением установленных сроков, то перерыв между инъекциями не должен превышать 12-13 месяцев. Вакцинацию выполняют с учетом ранее введенных доз. Срок отсчета выполнения последующей манипуляции осуществляется от даты введения последней дозы.Максимальный интервал между прививками — срок в 45 дней, но если по каким-то причинам введение препарата было пропущено, то вторая и третья прививка делается по мере возможности — делать лишнюю вакцинацию не нужно. Если первое введение вакцины АКДС было произведено позже, чем в три месяца, то ревакцинацию проводят через 12 месяцев после третьей инъекции.

В 7 и 14 лет детям проводят ревакцинацию против столбняка и дифтерии, применяя вакцину АДС-М или её аналоги. Такие ревакцинации необходимы, чтобы поддерживать количество антител и стойкость иммунитета на должном уровне.

Взрослым ревакцинацию против столбняка и дифтерии делают каждые десять лет.

Вопрос:

Мне 30 лет. В начале 2016г. я был на приеме у участкового терапевта в поликлинике по месту жительства. Выяснилось, что у меня не сделаны некоторые прививки, на которые меня направил мой терапевт (прививки делались в той же поликлинике). Я сделал все необходимые прививки, кроме одной. Мой вопрос связан именно с ней. К сожалению, я не могу вспомнить, как именно она называлась. Но данная прививка должна делаться в 3 этапа (под лопатку), с определенным интервалом. То есть первый укол в день обращения, второй кажется через месяц, а третий кажется через 5 месяцев (точные интервалы я не помню, могу ошибаться, но они примерно такие, как я указал). К сожалению, я сделал лишь 2 инъекции, т.е. в день обращения, и через месяц, а вот третью инъекцию я сделать не смог, по причине загруженности и работы. В связи с этим у меня вопрос, скажите пожалуйста, не отразится ли негативно на моем здоровье недоделанная прививка? Могут ли быть негативные последствия для меня? И также хотел уточнить, чтобы доделать эту прививку мне нужно будет начинать с начала, т.е. с первого укола, или же можно сделать только третий?

Ответ

Отвечает Полибин Роман Владимирович

Скорее всего, Вы имеете ввиду прививку АДС-м (от дифтерии и столбняка). Если человек не был привит в детстве, тогда график вакцинации меняется. Курс вакцинации должен состоять из 2 прививок с интервалом между ними от 30 до 45 дней. Первую ревакцинацию делают спустя 6–9 месяцев однократно, то есть её Вам как раз сейчас и пришло время сделать. Вторую ревакцинацию назначают через 5 лет. Все последующие введения вакцины от дифтерии взрослые люди проходят каждые 10 лет.

Вопрос:

11.11.2015 родилась дочь. В роддоме ребенку были сделаны 2 прививки БЦЖ и от гепатита В(производство Москва), вторая прививка от гепатита сделана ровно через месяц (производство Москва), третья прививка пропущена и не сделана. Сейчас ребенку 1 год, планируем ставить пентаксим 3 раза через полтора месяца. Наш график сместился. Подскажите, когда и как поставить третью прививку от гепатита В? Когда можно поставить прививку от пневмококковой инфекции? Когда ставить прививку от краснухи, кори и паротита? Что с чем можно совместить?Как будет выглядеть дальнейший наш график прививок?

Ответ

Отвечает Полибин Роман Владимирович

График, по которому делаются прививки от гепатита желательно соблюдать в точности. Изменения допускаются с одобрения врача, однако, если прививка задерживается более чем на три месяца, вся схема начинается заново, с первой прививки.

Согласно национальному календарю профилактических прививок, при сдвиге сроков вакцинации, допускается введение вакцин, применяемых в рамках Национального календаря профилактических прививок, в один день разными шприцами в разные участки тела. Исключением является только вакцина для профилактики туберкулеза!Если вакцины вводят не в один день, то интервал между прививками должен составлять не менее месяца.

Схема вакцинации «Пентаксим» применяется следующая.

1)Интервалы между дозами сохраняются и составляют 1,5–1,5 месяца. И 12 месяцев для ревакцинации.

2)После года жизни первая доза вводится полностью, а вторая, третья и четвёртая (ревакцинирующая) — без ХИБ-компонента. Вакцина «Пентаксим» без ХИБ-компонента может быть применена до шестилетнего возраста. В случае, когда нет необходимости использовать ХИБ-компонент, можно применить вакцину «Тетраксим». Это биопрепарат того же производителя, аналогичный «Пентаксим», но без гемофильной составляющей.

Вакцинация детей, которым иммунопрофилактика против пневмококковой инфекции не была начата в первые 6 месяцев жизни, проводится двукратно с интервалом между прививками не менее 2 месяцев.

Вакцинация от кори, краснухи и паротита. Если прививку надолго отложили— её делают максимально приближенной к графику. В этом случае промежуток между введением вакцины и проведением ревакцинации должен составлять не менее 4 лет.

Вопрос:

У моего ребёнка была нарушена схема вакцинации от Гепатита В.

1вакц: 25.12.2014г.(в роддоме), 2вакц: 20.09.2016г. На данный момент (17.11.2016г.) врач отказался делать вакцинацию и сказала прийти в феврале 2017г. на 3 вакцинацию. Вопрос в следующем, нужно ли начинать вакцинацию от Гепатита В заново и по какой схеме?

Ответ

Отвечает Харит Сусанна Михайловна

Продолжайте прививаться против вирусного гепатита В. Вакцинация в 2014 года уже не засчитывается, но вакцинацию, выполненную в сентябре 2016, продолжите по следующей схеме: 2 В против гепатита В выполните в ближайшие сроки, 3 вакцинацию против гепатита В — с 20.03.17.

Вопрос:

Дочке 6 лет. Ранее вакцинация не проводилась. Имеем носительство стрептококка (кашель с мокротой и зеленые сопли чередуются каждые 3 недели). Можно сделать прививку или не стоит?

Ответ

Отвечает Харит Сусанна Михайловна

Противопоказанием для вакцинации является наличие острого заболевания или обострение хронической патологии на момент вакцинации. Если на момент вакцинации ребенок здоров в течение месяца – прививаться можно. Перед прививкой необходим клинический анализ крови и общий анализ мочи (если это первая вакцинация и ребенок часто болеющий).

В связи с чем ребенок так часто болеет, посещает ли детский сад и всегда ли наблюдались такие частые респираторные заболевания? Носительство какого стрептококка имеет место быть? В связи с чем не проводилась вакцинация? Эти вопросы недостаточно понятны для нас. Вам стоит обсудить их с вашим лечащим врачом.

Вопрос:

Прививки акдс сделали по графику 2 вакцины, но третью пропустили на 2 года сейчас нам 2.7 можно ли нам ее сделать?

Ответ

Отвечает Харит Сусанна Михайловна

Все прививки, сделанные вами, учитываются. Вы можете сделать сейчас 3 вакцинацию АКДС с ревакцинацией через 6 месяцев. Главное, успеть привиться от коклюша до 4 лет, поскольку в нашей стране после 4-х лет делают АДС (без коклюшного компонента). Существует зарубежная вакцина для вакцинации детей старшего лет, подростков и взрослых, содержащая коклюшный компонент — Адацел, она зарегистрирована, но появится не раньше лета 2017 г.

Вопрос:

График прививок у нас нарушен, так первая прививка от полиомиелита (отечественная) сделана в 4 месяца, вторая такая же в 7 месяцев, в 1 г. 2 мес. в травм пункте была сделана экстренная прививка от столбняка, после чего в 1 г. и 3 мес. была сделана прививка Пентаксином (АКДС +полиомиелит +гемофильная инф.). Подскажите, пожалуйста, схему дальнейших прививок АКДС — чем лучше прививаться? И полиомиелита — какую лучше сделать, уколы или капли?

Ответ

Отвечает Харит Сусанна Михайловна

Не очень понятно, какой на данный момент возраст ребенка. Предполагаем, что около 1.5 лет. Тогда вы можете привиться вновь Пентаксимом, через 2 месяца- вновь Пентаксим, через 1,5 месяца Инфанрикс или АКДС. В целом у ребенка должно быть пять прививок против полиомиелита (в любом возрате) и 4 введения коклюшных вакцин (к 4 годам).

Вопрос:

Ребенку 9 месяцев. На данный момент поставлены только БЦЖ в роддоме и 2 гепатита В. С 3 месяцев мед.отвод у невролога и потом низкий гемоглобин (90). Сейчас разрешили прививаться. Как нам лучше спланировать наш график? Хотим ставить Пентаксим.

И еще — знаю, что корь, краснуха, паротит ставится в 12 мес. Но у нас в соседнем регионе вспышка кори. Нет ли необходимости срочно нам привиться?

Ответ

Отвечает Харит Сусанна Михайловна

Вакцина против кори до 12 месяцев неэффективна, в случае контакта вводится иммуноглобулин, содержащий готовые антитела. Также обязательна вакцинация против кори всех в окружении ребенка. График прививок: для того, чтобы вакцинация против гепатита В не пропала, сделайте сначала Инфанрикс Гекса или Пентаксим + 3 вакцинации против Гепатита В, затем 2 Пентаксима с интервалом в 1,5 месяца. Рекомендуем также привиться против пневмококковой инфекции.

Вопрос:

Добрый день. В связи со сложными родами (недоношенность, отслойка плаценты, реанимация и т. д.) у ребенка имелся медотвод от прививок по неврологии. В 1 год была сделана первая прививка от полиомиелита имоваксполио 1 декабря 2015 года. С февраля 2015 года ежемесячно болели, а потом получили медотвод от прививок до октября 2016 года.

Когда теперь нам делать вторую прививку от полиомиелита? Можно ли делать после 1 декабря 2016 года? Может ли пропасть первая вакцинация? Как сказывается такой долгий интервал между прививками?

Ответ

Отвечает Харит Сусанна Михайловна

Да, вы можете делать прививки после 1- го декабря. Продолжайте прививаться против полиомиелита, сделайте 2 прививки с интервалом в 1,5 месяца, с ревакцинацией через 6-9 месяцев, ваша первая вакцинация засчитывается. При нарушении интервалов между прививками иммунитет вырабатывается медленнее, но при завершенном курсе прививок ребенок будет полностью защищен.

Вопрос:

В роддоме не желали прививки. Был мед.отвод на месяц. В 2 месяца через манту сделали бцж-м. 28.07.16 сделали гепатит первую. Хотели по схеме 0-1-6, но в связи с переездом пропустили вторую гепатит, которая через месяц должна была быть. Потом простыли. В итоге 3.5 месяца прошло. Как сейчас быть?!

Ответ

Отвечает Харит Сусанна Михайловна

Делайте вторую против гепатита В, а с 28.01.17 — третью против гепатита В.

Вопрос:

Ребёнку 4 года. Первую прививку поставили около 2х лет Пентаксим. Через 3 месяца поставили 2-ую вакцину Пентаксим без гемофильной. Через 8 месяцев поставили Инфанрикс и прокапали полиомиелит. Далее ещё два раза капали полиомиелит, те полиомиелит закончили. В мае должны были ставить ревакцинацию , но Инфанрикса нет по сей день. Подскажите чем сделать ревакцинацию

Ответ

Отвечает Харит Сусанна Михайловна

В возрасте после 4 лет прививаются только АДС (без коклюшного компонента), необходимо сделать АДС.

Вопрос:

В прививочном календаре указано, что прививку от гемофильной инфекции начинают ставить в 3 мес. детям, находящимся в группе риска. Нам предложили провести данную вакцинацию впервые в 8 мес. — я отказалась, т.к. мы не относимся к группе риска. В 10 мес. нам вновь предложили, сказав, что я смотрела не актуальную редакцию прививочного календаря. В результате первую прививку от гемофильной инфекции нам поставили уже в 1 год 4 мес. Когда нам необходимо ставить следующую прививку от гемофильной инфекции и сколько раз её ещё ставить?

Ответ

Отвечает Харит Сусанна Михайловна

При начале вакцинации в возрасте от 1 года до 5 лет необходима однократная инъекция. Так, что вы уже полностью привиты.

Следует ли в условиях чрезвычайных ситуаций проводить противостолбнячную вакцинацию людей, получивших травмы?

Ответ: В условиях чрезвычайных ситуаций необходимо рассмотреть вопрос об иммунизации против столбняка лиц, получивших тяжелые травмы.

Поскольку столбняк является одним из осложнений при ранениях и травмах, чрезвычайно важно предоставлять пациентам соответствующую хирургическую и медицинскую помощь для лечения инфицированных открытых ран. Открытые раны следует считать инфицированными и воздерживаться от немедленного наложения швов. Важнейшим вопросом является очистка раны и удаление мертвой ткани и инородных тел, для чего, в зависимости от размера раны, может потребоваться хирургическое вмешательство, которое следует проводить в надлежащих условиях. Рану необходимо перевязать стерильной повязкой и назначить пациенту отсроченное наложение первичного шва.

Вакцинация против столбняка обычно назначается жертвам стихийных бедствий, либо проводится в случае необходимости. Это объясняется рядом причин.

Типы противостолбнячных вакцин

  • АС – Столбнячный анатоксин
  • АКДС – Адсорбированная коклюшно-дифтерийно-столбнячная вакцина
  • АДС – Адсорбированная дифтерийно-столбнячная вакцина
  • АДС-м – Адсорбированная дифтерийно-столбнячная вакцина с уменьшенным содержанием дифтерийного анатоксина
  • Во-первых, это заболевание не передается от человека к человеку.
  • Во-вторых, во многих частях мира большинство людей уже в той или иной мере защищены от столбняка за счет иммунизации.

Вкратце, в условиях чрезвычайных ситуаций для профилактики столбняка могут быть приняты следующие меры:

  • Тщательная очистка ран, как глубоких, так и поверхностных, может существенным образом снизить риск заражения столбняком. Важной также является защита обработанных ран от повторного заражения посредством наложения повязок и/или применения местных дезинфицирующих средств.
  • По возможности все лица, занятые при расчистке или разборе завалов, должны носить защитную одежду и закрытую обувь.
  • Если возможно, следует проводить иммунизацию вакциной АС всех лиц, участвующих в разборе завалов и ведении строительных работ. Лицам, не проходившим ранее курс иммунизации АС или относительно прошлой иммунизации которых уверенности нет, необходимо ввести повторную дозу вакцины через 4-8 недель после первой. Одна доза вакцины не обеспечит защиту тем, кто ранее не был вакцинирован против столбняка, а, скорее, подготовит организм к формированию иммунитета после получения повторной дозы вакцины, которая должна вводиться по меньшей мере через 4 недели.
  • По возможности следует вакцинировать АС-анатоксином всех внутренне перемещенных лиц, не получивших ранее по меньшей мере трех доз вакцины.
  • Ввиду того, что в районах, затронутых стихийным бедствием, отмечается повышенный риск заражения столбняком и что информация о прививочном статусе большинства людей в таких условиях отсутствует, целесообразным представляется проведение иммунизации вакцинами, содержащими АС-анатоксин, даже если имеющееся количество вакцины не позволяет вводить более одной дозы.
  • В отношении пациентов с глубокими ранами может быть необходимым использование противостолбнячного иммуноглобулина.

В условиях чрезвычайных ситуаций в районах с низким уровнем противостолбнячной иммунизации населения особое внимание следует уделять профилактике столбняка у матерей и новорожденных посредством иммунизации женщин детородного возраста и улучшения санитарно-гигиенических условий, в которых принимаются роды. Кроме того, необходимо в кратчайшие сроки восстановить рутинную иммунизацию женщин и детей.

Ссылки по теме

Десять прививок, которые делают взрослым

Фото: Итар-Тасс

График профилактических прививок существует не только для детей, но и для взрослых. Игнорируя рекомендации врачей, взрослые рискуют потерять здоровье и работоспособность.

Многие взрослые и забыли, когда делали прививку последний раз. Распространено заблуждение, что прививки по графику нужны только детям. Однако врачи напоминают, что с годами действие некоторых прививок, сделанных в детстве, проходит. И для взрослых также есть рекомендованный график прививок и реиммунизации. Прививки для взрослых, которые Вам необходимы:

1. Столбняк, дифтерия, коклюш. Эту прививку нужно делать каждые 10 лет. Беременным женщинам, которые делали прививку больше 10 лет назад, рекомендуется сделать прививку во втором или третьем триместре.

2. Человеческий папилломавирус (HPV). Иммунизация проводится в три этапа женщинам в возрасте 11-26 лет. Стоит отметить, что несмотря на то, что этот вирус в первую очередь угрожает женскому здоровью (провоцирует рак шейки матки), в некоторых странах эта прививка обязательна не только для девушек, но и для юношей (чтобы последние не становились носителями вируса).

3. Ветряная оспа (ветрянка). Прививка рекомендована взрослым, не имеющих иммунитета к вирусу ветряной оспы. Известно, что взрослые переносят это заболевание намного тяжелее, чем дети. Стоит помнить, что даже если вы болели ветрянкой в детстве, с возрастом приобретенный иммунитет может ослабнуть. Проверить наличие иммунитета можно сделав специальные пробы.

4. Опоясывающий лишай. Заболевание вызывается все тем же вирусом ветряной оспы. Прививка рекомендуется взрослым от 60 лет и старше.

5. Корь, эпидемический паротит, краснуха. Эта прививка входит в обязательный детский график прививок. Но она также рекомендована тем взрослым, которые не получили по крайней мере одну дозу этой прививки в детстве и никогда не болели корью, свинкой или краснухой.

6. Грипп. Ежегодная вакцинация в первую очередь рекомендована взрослым от 50 лет и старше. Более молодым людям прививку от гриппа делают по определенным медицинским, профессиональным и социальным показаниям. Если вы регулярно пользуетесь общественным транспортом, а на работе вы постоянно общаетесь с разными людьми, то вы попадаете в группу тех, кому эта прививка показана. На сегодняшний день многие предприятия оплачивают своим работникам вакцину против гриппа.

7. Пневмококк. Прививку рекомендуется делать взрослым от 65 лет, при условии, что они много курят, или часто бывают в больницах. Если вы моложе, но часто болеете бронхитами и воспалением легких, посоветуйтесь с врачом — вероятно, вам поможет именно эта прививка. Через пять лет будет необходима повторная вакцинация.

8. Гепатит A. Основными показаниями к этой прививке являются хронические заболевания печени, алкоголизм и наркомания. Также от гепатита А обязательно должны прививаться медицинские работники.

9. Гепатит B. К показаниям, которые уже перечислены для вакцинации против гепатита А, добавляется «частая смена сексуальных партнеров» (как деликатно формулируют медики) и лечение инъекциями. Некоторые врачи считают, что прививку против гепатита В нужно делать всем взрослым. Ведь теоретически, заразиться можно у стоматолога, гинеколога, или во время хирургической операции.

10. Менингококк. Прививаться рекомендовано тем взрослым, которые вынуждены регулярно находиться в больших коллективах. Например, живущим в общежитиях студентам и военнослужащим. Жители таких мегаполисов, как Москва и Петербург, больше других россиян рискуют «подхватить» менингит.

Конечно, перед тем, как провести вакцинацию, необходимо проконсультироваться с врачом, сдать необходимые анализы и убедиться, что в день прививки вы абсолютно здоровы.

Выделите фрагмент с текстом ошибки и нажмите Ctrl+Enter

Календарь прививок — причины, диагностика и лечение

  • Врачи
  • Лечение
  • Диагностика
  • Статья обновлена: 18 июня 2020

В настоящее время в России действует Российский календарь профилактических прививок, утвержденный приказом Минздрава РФ №229 от 27.06.2001 г.

Что это?

Календарь прививок — это список обязательных прививок, в котором указывается, какую вакцину и в каком возрасте следует вводить. В настоящее время в России действует Российский календарь профилактических прививок, утвержденный приказом Минздрава РФ №229 от 27.06.2001 г.

Национальный календарь прививок предусматривает вакцинацию против следующих заболеваний:

  • Гепатит В — инфекционное вирусное заболевание, поражающее печень. Может приобретать хроническое течение с формированием цирроза печени.
  • Туберкулез — инфекционное бактериальное заболевание, чаще всего поражает лёгкие.
  • Полиомиелит — острое инфекционное заболевание вирусной природы, сопровождается формированием необратимых парезов и параличей.
  • Дифтерия — острое инфекционное бактериальное заболевание при котором поражаются дыхательные пути, сердце, нервная система, почки и другие органы. До применения противодифтерийной вакцины болезнь в большинстве случаев заканчивалось летально.
  • Коклюш — острое инфекционное бактериальное заболевание, сопровождающееся приступообразным кашлем.
  • Столбняк — острое инфекционное бактериальное заболевание, характеризующееся поражением центральной нервной системы. Вызывает развитие судорог и удушья (асфиксии).
  • Корь — острое вирусное заболевание, проявляющееся симптомами интоксикации (повышение температуры и т.д.), сыпью и поражением слизистой оболочки носа, глотки. Корь может сопровождаться тяжелыми осложнениями.
  • Краснуха — вирусная инфекция, для которой характерно появление сыпи и увеличение лимфатических узлов. Особенна опасна краснуха в первой половине беременности, поскольку может приводить к выкидышу или тяжелым порокам развития плода.
  • Паротит (свинка) — острое вирусное заболевание, при котором поражаются слюнные железы, нервная система. У мальчиков часто вовлекаются яички, что приводит к бесплодию.

Российский календарь профилактических прививок

При использовании инактивированных вакцин для создания защитного иммунитета недостаточно одного укола. Обычно требуется проведение курса вакцинации, состоящего из 2-3 уколов, с последующей ревакцинацией, то есть дополнительной «подпиткой» иммунитета.

  • Самая первая вакцинация проводится, когда новорожденному исполняется 12 часов. Ребенку делают прививку от гепатита В.
  • С 3 по 7 день ребенка прививают от туберкулеза вакциной под названием БЦЖ..
  • Как только ребенку исполняется один месяц (30 дней), проводят вторую вакцинацию от гепатита В.
  • В три месяца ребенка нужно вакцинировать сразу против четырех болезней: коклюша, дифтерии, столбняка, полиомиелита. Однако на самом деле ему сделают только две прививки (или даже одну — «тетракок»): первая — против дифтерии, столбняка и коклюша (например, вакциной АКДС) и вторая — против полиомиелита.
  • В четыре с половиной месяца делают то же самое, что и в три.
  • В шесть месяцев то же самое, что и в четыре с половиной. Кроме того, в третий раз вакцинируют от гепатита В.
  • Когда ребенку исполняется один год , пора делать прививку от кори, краснухи и паротита (свинки) (одна вакцина).
  • В полтора года делают ревакцинацию от коклюша, дифтерии, столбняка, а также вводят вакцину от полиомиелита.
  • В 20 месяцев — очередная ревакцинация от полиомиелита.
  • Следующие прививки нужно делать только в 6 лет. Ребенку вводят очередную порцию вакцины против кори, краснухи и паротита.
  • В 7 лет : первая ревакцинация против туберкулеза (БЦЖ), вторая ревакцинация против дифтерии и столбняка (АДС).
  • В 13 лет прививки делают выборочно. Если ребенку не были сделаны вовремя прививки от гепатита В, самое время заняться этим сейчас. Прививку от краснухи в 13 лет делают только девочкам.
  • В 14 лет — очередная ревакцинация против дифтерии и столбняка, полиомиелита, туберкулеза.
  • Взрослые должны ревакцинироваться от дифтерии и столбняка каждые десять лет с момента последней прививки.

Чем прививают?

Иммунизация в рамках национального календаря профилактических прививок проводится вакцинами отечественного и зарубежного производства, зарегистрированными и разрешенными к применению в установленном порядке в соответствии с инструкциями по их применению.

Вакцина АДС-М — многопрофильная клиника Чудо-Доктор в Москве

Наличие вакцин уточняйте у администраторов клиники.

Дифтерия (греч. διφθέρα — кожа), устар. дифтерит — инфекционное заболевание, вызываемое бактерией Corynebacterium diphtheriae (бацилла Лёффлера). Чаще всего поражает ротоглотку, но нередко затрагивает гортань, бронхи, кожу и другие органы.

Столбняк — это инфекционное заболевание, вызываемое бактерией Clostridium tetani, с острым характером течения и преимущественным поражением нервной системы.

Вакцина рассматривается как смесь компонентов, подвергнутых прохождению специальной обработки. Составляющие препарата не могут стать причиной тяжелой токсической реакции или спровоцировать инфекционное поражение. 

Прививка АДС-М подстегивает иммунную систему на соответствующий ответ, в котором и заключается принцип ее действия. В результате человек обретает невосприимчивость к таким заболеваниям, как дифтерия и столбняк.

Симптомы инфекционного заболевания

Перечисленные инфекции относятся к разряду бактериальных патологий острого характера. Заболевший представляет угрозу для окружающих. В ситуации со столбняком заразиться реально и от животных. Дифтерия локализуется в области верхних дыхательных путей.

К числу клинических симптомов относятся следующие признаки:

  • Увеличение температуры на фоне слабости и измененных размеров лимфатических узлов;
  • Болезненные ощущения в горле, тканевый отек в зоне шеи, трудности при глотании;
  • Гиперемия глоточной слизистой оболочки и увеличение миндалин;
  • Налет на них, захватывающий прилегающие ткани.

При наступлении осложнений страдает сердце, наступает мышечный шейный паралич, нарушается функционирование нервной системы. При особых обстоятельствах не исключается летальный исход.

Когда делают прививку от дифтерии?

Вакцина АДС-М считается комплексной. Ее введение допустимо через 3 месяца с момента рождения ребенка. Клиническая картина столбняка разнится с таковой при дифтерии.

Как правило, больные ощущают:

  • Температурное повышение и слабость;
  • Мышечную напряженность и спастичность лица наряду с аналогичными симптомами в остальными частях тела;
  • Проблемы с дыханием, заключенные в его затруднении;
  • Дыхательные спазмы, приводящие к смертельному исходу.

Рассматриваемые заболевания наносят ущерб нервной системе. Их возбудители невосприимчивы к лечению даже сильнейшими антибактериальными средствами. Облегчить протекание инфекции или предотвратить ее поможет вовремя сделанная АДСМ прививка.

Она показана не только в детском возрасте. Возрастная принадлежность не служит препятствием для принятия профилактических мер, направленных на отведение угрозы заражения в любой момент.

Что за прививка АДС-М?

Формой выпуска вакцины является суспензия. Для нее характерен желто-белый оттенок. Все ампулы вмещают 1 мл вещества, соответствующий удвоенной дозе анатоксина.

Инструктивные материалы рекомендуют применять прививку, если:

  • Планируется профилактика дифтерии и столбняка в детском возрасте с 6 лет;
  • Необходимо вакцинировать взрослых, с момента последнего прививания которых прошло 20 лет;
  • Требуется заменить АКДС и АДС, основываясь на детских реакциях и осложнениях;
  • Нужно привить 4-летних детей, не получивших в свое время АКДС.

Прививка АДС-М взрослым и детям показана каждые 10 лет. Она формирует иммунитет на долгие годы.

Куда делают прививку АДС-М?

В соответствии с обновленными рекомендациями практикуется внутримышечное введение препарата в области передненаружной части бедра. Возможно подкожное глубокое инъецирование в подлопаточную зону.

Противопоказания для применения АДС-М

АДС-М прививка детям и взрослым не делается при наличии противопоказаний постоянного или временного свойства.

К числу первых относятся:

  • Яркая реакция на вакцинацию указанным препаратом в прошлом;
  • Осложнения после его введения.

Ко второй группе принадлежат:

  • Беременность и кормление грудью;
  • Обострение хронических состояний;
  • Текущий инфекционный процесс;
  • Аллергическая реакция;
  • Продромальные явления с плохим самочувствием, суставной ломотой, слабостью.

Прививка разрешается спустя месяц после исчезновения признаков аллергии. В остальных случаях выдерживать паузу не требуется.

Люди, не привитые от инфекций, составляют группу риска. Они могут заразиться ими сами и инфицировать других, в особенности детей со слабым иммунитетом. Единовременная вакцинация АДС-М способна предотвратить возможную гибель человека от дифтерии и столбняка.

Преимущества вакцинации АДС-М в клинике «Чудо Доктор»

Обращение к врачу — терапевту с целью получения прививки сопровождается медицинским осмотром. В медицинском центре используются только те препараты, которые прошли сертификацию и подтвердили свое качество. Вакцина отличается высокой эффективностью с минимальными побочными явлениями.

Персонал клиники строго придерживается соблюдения правил при выполнении прививки от столбняка и дифтерии. Это обстоятельство исключает возникновение осложнений и побочных реакций на используемый препарат.

Специалисты «Чудо Доктор» помогут достигнуть максимального результата в деле сохранения здоровья. Вы не столкнетесь с неожиданными проблемами в будущем, вызванными такими серьезными заболеваниями.

Подумайте о своем здоровье – заблаговременно пройдите вакцинацию!


Какие прививки нужно делать детям в Кыргызстане

Опубликовано: 28 июня 2019Обновлено: 9 июля 2020

Поздравляем вас с рождением вашего долгожданного малыша! Это радостное, но вместе с тем очень ответственное событие для родителей. Ребенка хочется защитить от всех бед, особенно от опасных инфекций и болезней.

Сохранить здоровье малыша помогают прививки, особенно в первый год жизни. Некоторые родители ошибочно не доверяют вакцинации, поэтому пытаются отложить эту процедуру или вовсе отказываются от нее. Помните, что отложенная прививка детям первого года жизни – это дополнительный шанс для развития серьезного инфекционного заболевания!

Вакцина против гепатита В — ВГВ

В первые сутки малышу рекомендуется ввести вакцину против гепатита В, которая защитит малыша от возможного заражения от матери. Во время беременности каждая мама сдавала тест на антитела к вирусу гепатита В. Но даже при положительном результате малыш подвергается, пусть и небольшому, но риску заражения вирусом гепатита В от мамы при родах. Если не уделять должное внимание данной прививке и отложить ее на неопределенный срок, ничто не сможет остановить развитие инфекции в организме ребенка. Для того, чтобы в организме ребенка развить долговременную защиту от вирусного гепатита В необходимо повторить прививку 3 раза в возрасте 2, 3.5 и 5 месяцев.

Вакцина против туберкулеза — БЦЖ

Прививка против туберкулеза – вторая прививка первого года жизни в родильном доме. Ее введение ребенку в первые дни после рождения, позволяет избежать развития очень тяжелых форм туберкулеза, которые могут привести к летальному исходу. В Кыргызстане ситуация с заболеваемостью туберкулезом оценивается как неблагополучная. Болеют люди из всех социальных слоев. Высокая инфицированность и заболеваемость туберкулезом детей также свидетельствует о наличии источников инфекции среди населения.

Дифтерия, коклюш, столбняк, полиомиелит, пневмококк и гемофильная инфекция

Вот еще несколько инфекций, против которых делают защитные прививки на первом году жизни.

ПЕНТА (АКДС-ВГВ-ХИБ) — вакцинация против коклюша, дифтерии, столбняка, гемофильной инфекции и вирусного гепатита В, которая проводится ребенку в 2, 3.5 и 5 месяцев.

ПКВ – вакцинация против пневмококковой инфекции проводится в 2, 5 и 12 месяцев.

Пневмококковая инфекция – это причина большой группы заболеваний, которые проявляются различными гнойно-воспалительными процессами в организме. Инфекцию этой группы «подцепить» можно где угодно. Распространение микробов от человека к человеку происходит преимущественно воздушно-капельным путём при кашле или чихании, а также при контакте с предметами (контактный путь), которые соприкасались со слюной (ложки, чашки, игрушки и т.п.). Особенно маленькие дети со слабо развитым иммунитетом находятся в зоне риска.

ОПВ – вакцинация против полиомиелита проводится в 2, 3.5 и 5 месяцев.

Полиомиелит-это острое, вирусное инфекционное заболевание, поражающее центральную нервную систему. Источником заражения является больной человек и здоровые вирусоносители (наиболее частая форма). Заражение происходит через грязные руки, инфицированные продукты и воду. В настоящее время только с помощью проведения профилактических прививок можно предотвратить это заболевание.

КПК — вакцинация против кори, эпидемического паротита и краснухи, которую проводят, когда ребенку исполняется год.

Прививки детям первого года жизни – важная миссия родителей для сохранения здоровья их детей на протяжении многих лет

Для того, чтобы развить в растущем организме устойчивую и сильную иммунную систему, на протяжении нескольких лет проводится ревакцинация. Это необходимо для того, чтобы подготовить ребенка перед поступлением в дошкольные или школьные учреждения к встрече с новыми микробами и вирусами, многие из которых могут вызывать серьезные инфекционные заболевания. Детский сад и школа – это места, где инфекции могут распространяться очень быстро. Поэтому здесь становятся особенно опасны инфекции с высокой заразностью, такие как грипп, корь, краснуха, паротит (свинка), ветряная оспа. Вакцинация призвана помочь предотвратить их.

В 2 года проводится первая ревакцинация против коклюша, дифтерии и столбняка.

В 6 лет по плану вторая ревакцинация против дифтерии и столбняка, а также ревакцинация против кори, эпидемического паротита и краснухи.

В 11 лет проводится ревакцинация против дифтерии и столбняка.

Важно помнить о том, что определить правильную схему вакцинации и, при необходимости, дать соответствующие рекомендации может только детский врач, у которого ваш ребенок стоит в учете и наблюдается.

Смотрите подробнее по ссылке календарь прививок, согласованный Министерством Здравоохранения Кыргызской Республики.

Своевременная вакцинация — это ответственность каждого сознательного родителя перед своим ребенком!

Источники информации:
Информационные материалы, составленные сотрудниками РЦИ и одобренные Министерством здравоохранения КР 2018г.

Поделиться в социальных сетях!

Какие прививки необходимо делать взрослым

Зачем взрослым прививки от детских болезней?

Детскими болезнями обычно называют корь, дифтерию, ветрянку и большинство болячек, от которых делают прививки. На самом деле они вовсе не детские — с возрастом ничего не меняется.

Просто все эти болезни легко подхватить. До начала массовой вакцинации люди заражались, как только сталкивались с возбудителями. Происходило это в раннем возрасте, а дальше заболевшие либо умирали, либо приобретали активный иммунитет, который защищал их. Вот и казалось, что болеют только дети.

Сейчас не надо рисковать жизнью ради иммунитета — есть прививки. Но если вам их не делали или делали слишком давно, вы в группе риска.

Ольга Владимировна Ширай

врач-эпидемиолог, начальник эпидемиологического отдела СПб ГБУЗ «Елизаветинская больница»

В настоящее время никто не будет предупреждать человека о сроках иммунизации: необходимо самому узнать правильное время и возраст ревакцинации.

Многие взрослые не делают ревакцинацию, но всё равно не болеют благодаря иммунитету. У кого-то он остался после болезни, у кого-то после прививки (даже если о ней все забыли), других защищает коллективный иммунитет — эпидемиям просто негде разгуляться, если большинство привито. Ревакцинация нужна, чтобы не болеть и не провоцировать эпидемии.

Как узнать, какие прививки мне делали?

В теории все прививки записываются в карточку или прививочный сертификат, а карточки с данными кочуют с человеком из поликлиники в поликлинику.

На практике ничего этого нет. Даже если вы всю жизнь прикреплены к одной поликлинике, у вас не менялась прописка, все эти данные запросто могут потерять. Для всех остальных это квест «помню — не помню». Скорее всего, вы не помните.

Если это так, то для рождённых в России есть ориентир — национальный календарь прививок. Если прививка в нём есть, возможно, вы её получали. Тогда вам, скорее всего, нужна ревакцинация, ведь не все прививки действуют всю жизнь. Если прививки нет в национальном календаре, то нужно её сделать в любом случае.

Какие анализы покажут, что прививки были?

Если человек когда-то прививался, у него есть антитела к этому заболеванию. Это белки, которые атакуют бактерию или вирус, проникшие в организм. Они обозначаются как IgG. — иммуноглобулины типа G.

Проведите исследование крови на наличие антител к вирусному гепатиту В, дифтерии, столбняку, полиомиелиту (в отношении трёх типов вируса), кори, краснухе, эпидемическому паротиту, коклюшу. Для этого проводится реакция РПГА с соответствующим диагностикумом (дифтерия, столбняк, корь, паротит) или ИФА (коклюш, гепатит, краснуха).

Ольга Ширай

Чтобы иммунитет сработал, нужен определённый титр — количество этих самых иммуноглобулинов. Если титр маленький, надо вакцинироваться. Показатели для всех прививок свои, это обсуждается с врачом отдельно.

Но даже если вы сделаете прививку от болезни, к которой уже есть иммунитет, ничего особенного не произойдёт — введённые с вакциной агенты будут уничтожены.

Какими вакцинами можно прививаться?

В отношении вакцин действует правило «чем современнее, тем лучше», потому что исследователи постоянно работают над улучшениями. Новые вакцины хорошо переносятся, нередко защищают сразу от нескольких заболеваний.

Даже если вас прививали в детстве старыми вакцинами, смело можете делать ревакцинацию новыми — никакого конфликта не будет.

Мы перечислили вакцины, которые разрешено использовать в России . Чтобы узнать о них больше, нужно прочитать инструкцию и изучить противопоказания. Какие-то вакцины не встречаются в поликлиниках, а какие-то просто трудно найти.

Как прививаться от гепатита B?

Гепатит В — опасная вирусная инфекция, которая передаётся через кровь и половым путём. Она поражает печень, от неё нет специфических препаратов лечения. Протекать болезнь может тяжело и с осложнениями вплоть до смертельного исхода. От гепатита B каждый год умирает почти 700 000 человек в мире .

Вакцинация проводится детям и взрослым, ранее не привитым против вирусного гепатита B, по схеме 0–1–6 (первая доза — в момент начала вакцинации, вторая доза — через месяц после первой прививки, третья доза — через 6 месяцев с начала вакцинации).

Главное — сделать три прививки, чтобы больше не вспоминать об этом. Если вас начали прививать, но схему не закончили, то иммунитет будет нестойким и никто точно не скажет, сколько он продержится.

Вакцины: «Эувакс В», «Регевак В», «Энджерикс В».

Как прививаться от дифтерии, коклюша и столбняка?

Детям дают вакцину, которая защищает сразу от трёх заболеваний. После 26 лет нужно проходить ревакцинацию как минимум от двух из них каждые 10 лет.

  • Дифтерия — болезнь, которая поражает горло и бронхи вплоть до того, что человек не может дышать. Протекает тяжело, с высокой температурой и поражением внутренних органов. Возбудитель инфекции — дифтерийная палочка — токсичен, поэтому нередко появляются осложнения. Болезнь смертельно опасна.
  • Столбняк поражает нервную систему, заразиться им можно в любое время, если инфекция попадёт в ранку (например, с занозой). Из-за столбняка начинаются судороги, а при плохом развитии болезни человек умирает, потому что отключаются нервы, ответственные за дыхание.
  • Коклюш поражает дыхательную систему, отличается характерным кашлем. Протекает тем тяжелее, чем младше больной.

Взрослых по национальному календарю прививают только от дифтерии и столбняка (вакциной АДС-м). Коклюшный компонент в вакцину не входит, поскольку для взрослого болезнь не так страшна, как для ребёнка. В коклюше в любом возрасте ничего хорошего нет, поэтому рекомендуем купить вакцины с коклюшным компонентом, чтобы защититься от всего сразу.

Если вакцинация в детстве не проводилась, то необходимо сделать три прививки: первые две дозы вакцины вводятся с интервалом в один месяц, третья — через год после введения второй дозы. Затем ревакцинация проводится также один раз в 10 лет.

Ольга Ширай

Даже если вам что-то прививали в школе, после 26 лет нужна ревакцинация как минимум от дифтерии и столбняка.

Американская педиатрическая ассоциация рекомендует прививаться от коклюша беременным женщинам, чтобы передать антитела новорождённому.

Вакцины: против дифтерии и столбняка — «АДС-М»; против дифтерии, столбняка и коклюша — «Адасель».

Дмитрий Малых

врач-педиатр, невролог, член Союза педиатров России.

«Адасель» — это единственная в Европе вакцина против коклюша, лицензированная для вакцинации взрослых. Препарат «АДС-М» включён в государственную страховку и доступен бесплатно по полису ОМС. «Адасель» оплачивается рядом страховых программ добровольного медицинского страхования, также её можно получить за наличный или безналичный расчёт в частных клиниках страны. Оба препарата доступны на рынке.

Как прививаться от кори, краснухи, паротита?

От этих заболеваний детей тоже прививают одновременно.

  • Корь — высокозаразная болезнь, которая опасна осложнениями, такими как энцефалит или пневмония.
  • Краснуха представляет особенную опасность для беременных, потому что поражает плод.
  • Паротит, он же свинка, опасен, потому болезнь нередко заканчивается осложнениями: поражаются железы, почки, головной мозг.

Прививку, если она была когда-то давно, необходимо повторять в возрасте 22–29 лет (в зависимости от сроков последней ревакцинации), а затем через каждые 10 лет.

В медицинской литературе можно встретить данные о том, что иммунитет после прививки от кори и эпидемического паротита сохраняется в течение 20–30 лет. Поэтому нет необходимости вводить трёхкомпонентную вакцину каждые 10 лет, а можно делать прививку только от краснухи, поствакцинальная защита от которой существует только 10 лет. Однако это спорный вопрос. Спустя 10 лет после прививки защита от кори и паротита также может ослабнуть, поэтому рекомендуется использовать для ревакцинации вакцину, содержащую все три вируса.

Ольга Ширай

Взрослые люди, которые не переболели этими инфекциями в детстве и не прививались, для формирования иммунитета получают две дозы вакцины с интервалом между введениями в месяц, затем ревакцинация проводится также один раз в 10 лет.

Вакцины: M-M-P II, вакцина коревая-паротитная культуральная живая, вакцина против краснухи культуральная живая.

Как прививаться от ветрянки?

Прививка от ветряной оспы была внесена в национальный календарь совсем недавно, она выполняется по желанию. Взрослому человеку она может быть сделана в любом возрасте, если в детстве он не переболел ветрянкой или не был привит.

Исследования показали, что иммунитет после прививки сохраняется дольше 30 лет, поэтому программы ревакцинации не предусмотрены (прививка от ветрянки делается один раз в жизни).

Вопреки распространённому мнению, ветрянкой не лучше переболеть. Во-первых, взрослые переносят болезнь тяжелее, чем дети. Во-вторых, вирус, который вызывает ветряную оспу, остаётся в организме навсегда и может появиться снова в виде опоясывающего лишая.

Женщинам, которые не болели ветрянкой и планируют беременность, тоже следует сделать прививку от этого заболевания, так как заражение во время беременности (особенно на ранних сроках) может привести к развитию пороков у плода и даже выкидышу.

Ольга Ширай

Прививку от ветрянки можно сделать и для профилактики заболевания, если имел место контакт с заболевшим человеком. ВОЗ считает эту меру эффективной, если вакцинация проведена не позднее 72 часов после контакта здорового человека с больным.

Вакцина: «Варилрикс».

Как прививаться от полиомиелита?

Вирусы полиомиелита страшны осложнениями: один из 200 переболевших получает осложнения в виде параличей. Прививаться нужно, если нет иммунитета и вы собираетесь в страну, где болезнь встречается часто.

Прививка делается в три этапа. Лучше использовать инактивированную вакцину — ту, которая в уколах, она намного безопаснее капель в рот.

Вакцины: «Имовакс Полио», «Полиорикс», «Тетраксим».

Как прививаться от гемофильной инфекции?

Гемофильная инфекция вызывает тяжёлые формы менингита, пневмонии, иногда приводит к сепсису. Инфекция плохо поддаётся антибиотикам.

Среди детей вакцинируют только тех, кто в группе риска по состоянию здоровья. Это относится и к взрослым: прививают пожилых, людей, контактирующих с больными, людей с хроническими заболеваниями.

Вакцины: «Акт-ХИБ», «Хиберикс».

Как прививаться от папилломавируса?

Некоторые типы папилломавируса (ВПЧ) представляют опасность для женского здоровья, вызывая рак шейки матки, остроконечные кондиломы и некоторые другие заболевания.

Рекомендуется вакцинация девочкам и женщинам в возрасте от 9 до 26 лет, желательно до начала половой жизни (потому что с её началом резко увеличивается риск заразиться от партнёра). Прививку можно сделать и в более позднем возрасте до 45 лет.

Ольга Ширай

Мужчинам тоже можно делать эту прививку, чтобы не дразнить вирус, вызывающий рак (не только шейки матки, но и других органов), и не мучиться с кондиломами. Прививка делается в три этапа по инструкции.

Вакцины: «Гардасил», «Церварикс».

Сейчас появился ещё один препарат против вируса папилломы человека. Торговое название — «Гардасил 9». В отличие от «Гардасила», который защищает от четырёх серотипов вируса папилломы человека, «Гардасил 9» защищает от девяти серотипов ВПЧ. Вакцина доступна в США и ряде европейских стран.

Дмитрий Малых

Как прививаться от пневмококка?

Взрослым прививка делается по желанию. Пневмококковая инфекция, как правило, присоединяется к другим болезням и является осложнением. Она вызывает менингит, средний отит, синусит, пневмонию.

Рекомендуется вакцинация людям, входящим в группу повышенного риска заражения, а также тем, у кого заболевания, вызываемые пневмококком, протекают наиболее тяжело и вызывают осложнения:

  • взрослым старше 65 лет;
  • людям, часто контактирующим с возможными носителями инфекции;
  • работникам детских дошкольных, школьных учреждений, домов-интернатов;
  • тем, у кого есть хронические заболевания органов дыхания, печени, сахарный диабет;
  • людям, страдающим иммунодефицитами;
  • больным, у которых повышен риск возникновения менингита (после черепно-мозговых травм, нейрохирургических вмешательств на позвоночнике).

Вакцины: «Пневмо-23», «Превенар 13».

Как прививаться от менингококка?

Менингококк вызывает менингит, но особенный. Это всегда стремительная инфекция, потенциально смертельно опасная. Каждый случай заболевания — это чрезвычайная ситуация.

В России вакцинация проводится, если есть вспышка заболевания, а также среди тех, кто подлежит призыву на военную службу, и тех, кто выезжает в Африку и Азию.

В России в настоящий момент доступна вакцинация против менингококковой инфекции четырёх серотипов: A, C, Y, W-135. В большинстве европейских стран и странах Северной Америки доступна также вакцина против менингококка типа B. Торговое название препарата — «Бексеро». Я рекомендую приобрести этот препарат во время зарубежных поездок (после стабилизации ситуации с пандемией коронавируса).

Дмитрий Малых

Современные вакцины защищают сразу от нескольких подтипов заболевания. Достаточно одной прививки во взрослом возрасте.

Вакцины: «Менактра», «Менцевакс ACWY».

Какие ещё прививки стоит сделать?

Кроме перечисленных, есть ещё прививки по эпидемическим показаниям. Их делают в том случае, если где-то началась эпидемия или если человек в работе часто сталкивается с редкими заболеваниями. Это дело специалистов, но есть несколько прививок, которые стоит сделать, не дожидаясь эпидемий.

  • Клещевой энцефалит. Мы уже писали, кому, как и когда прививаться от клещевого энцефалита (начинайте заниматься вопросом в феврале, чтобы успеть сделать полный курс и выработать иммунитет до того, как проснутся клещи).
  • Грипп. Подробно про прививку от гриппа мы тоже уже писали. Прочитайте всё, что хотите узнать. Прививка — лучшая защита от гриппа. Стоит привиться до середины октября, чтобы встретить эпидемию во всеоружии.
  • Прививки для путешественников. Если вы собираетесь в страну, где часто возникают вспышки инфекций, то нужно перед поездкой сделать прививки. Обычно это гепатит А (от него можно привиться и просто для профилактики), жёлтая лихорадка. Всё зависит от страны, в которую вы решите ехать.

Что сделать прямо сейчас?

Чтобы гарантированно не болеть:

  1. Зайдите в поликлинику по месту жительства и спросите у терапевта, отметки о каких вакцинах есть в вашей карте.
  2. Сдайте анализы на антитела к тем заболеваниям, к которым это требуется.
  3. Уточните, есть ли в поликлинике вакцины, а также их названия.
  4. Найдите частный медицинский центр, у которого есть лицензия на проведение вакцинации.
  5. Узнайте, в каких аптеках продаются вакцины.
  6. Составьте вместе с врачом график вакцинации. Несколько вакцин можно вводить одновременно, вовсе не обязательно делать перерывы между разными препаратами. Всё зависит от инструкции к каждой конкретной вакцине.
  7. Сделайте прививки по этому графику.
  8. Не болейте.

Читайте также 💊🤔💉

Как ВИЧ и СПИД влияют на чернокожих

Во многих отношениях афроамериканцы пострадали от ВИЧ сильнее, чем любая другая расовая или этническая группа в Соединенных Штатах. Большая часть этого сообщества получает диагноз СПИДа и умирает от ВИЧ. В среднем афроамериканцы со СПИДом живут не так долго, как другие группы.

СПИД — основная причина смерти афроамериканцев, особенно молодых женщин. По данным CDC, в 2016 году у чернокожих женщин был диагностирован ВИЧ в четыре раза больше, чем у латиноамериканок или белых женщин.

Почему цифры такие высокие?

Недостаток осведомленности, недоверие и страх

Многие чернокожие люди могут быть ВИЧ-инфицированными и не знать об этом, поэтому они продолжают распространять вирус, но при этом заболевают. В 2014 году только 59% афроамериканцев, живущих с ВИЧ, принимали лекарства от этого заболевания. В 2015 году более половины умерших от ВИЧ составляли африканские ариканцы.

Некоторые афроамериканцы все еще ошибочно полагают, что ВИЧ — это болезнь белых геев. Из-за этого трудно рассказать им о ВИЧ или побудить их рассказать о своем ВИЧ-статусе.

Продолжение

Частично проблема может заключаться в сохраняющемся недоверии черного сообщества к правительственным источникам информации и исследований. Историческое исследование сифилиса Таскиги проводило вредоносные медицинские тесты на афроамериканцах без их ведома в течение 40 лет.

Стигма в отношении гомосексуализма может также заставить замолчать мужчин, имеющих половые контакты с мужчинами. Чернокожие мужчины «на низком уровне» занимаются сексом с мужчинами, но могут не рассказывать своим половым партнерам женщинам.

Незащищенный секс и ЗППП

Большинство афроамериканских мужчин и женщин заражаются ВИЧ, когда они не используют презерватив или другие средства защиты при половом акте с мужчиной.У чернокожего мужчины меньше шансов заразиться ВИЧ от женщины. И у черного мужчины больше шансов заразиться ВИЧ, занимаясь сексом с мужчиной.

Людям, у которых есть другое заболевание, передающееся половым путем (ЗППП), например гонорея, легче заразиться или передать ВИЧ, а уровень инфицирования ЗППП выше среди афроамериканцев.

Бедность

Афроамериканцы, как группа, чаще не застрахованы или застрахованы государством, чем белые. У них может не быть доступа к информации или такому количеству вариантов тестирования и лечения в связи с ВИЧ.Может быть труднее найти поддержку и услуги, чтобы помочь жить с ВИЧ или СПИДом и связанными с ним заболеваниями.

Женщины могут чувствовать, что они не могут защитить себя в сексуальных отношениях, когда они зависят от своего партнера в финансовом отношении.

Инъекционные наркотики

Совместное использование игл или шприцев с ВИЧ-инфицированными людьми является вторым по распространенности способом заражения ВИЧ-инфекцией афроамериканских мужчин и женщин. Он передает ВИЧ через контакт с кровью. Кайф также может привести к рискованному сексуальному поведению.

Наркотическая зависимость или наркомания могут усугубить денежные проблемы. И люди могут быть готовы обменять небезопасный секс на наркотики.

Как снизить влияние

Повышение осведомленности о том, что такое ВИЧ и что он делает для афроамериканского сообщества, — это начало. CDC и другие организации пытаются изменить представления о ВИЧ и СПИДе, чтобы больше чернокожих людей чувствовали себя в безопасности, говоря об этом, и чтобы они прошли тестирование и прошли лечение.

Продолжение

Чтобы избежать заражения ВИЧ, практикуйте безопасный секс.Спросите своего партнера об их ВИЧ-статусе. Используйте латексный презерватив и лубрикант на водной основе каждый раз, когда занимаетесь сексом. Если вы подозреваете, что у вас ЗППП, обратитесь к врачу и пройдите лечение.

Подумайте, следует ли вам проходить тестирование на ВИЧ всякий раз, когда вы проходите медицинский осмотр. Сделайте это частью сохранения здоровья. Не бойтесь попросить своего врача сделать тест на ВИЧ.

Всегда используйте чистые иглы и шприцы для инъекционных наркотиков; не используйте одно за другим. Попробуйте проконсультироваться или пройти курс лечения, чтобы помочь вам отказаться от наркотиков.

Тех, кого мы потеряли из-за эпидемии СПИДа

Дополнительный отчет Стива Гарбарино

Фото: Эйли: Bettmann / Getty Images. Аллен: Предоставлено Эвереттом. Арены: Фото Луи Монье / Gamma-Rapho через Getty Images. Хоккей с мячом: ого. Баркер: Томас Ианнакконе. Бенеш: Фото Брайана Ача / WireImage. Беннетт: © TopFoto / The Image Works. Бентли: Фото любезно предоставлено Ричардом Бентли. Бернд: Фотография Донны Энн Макадамс. Боташ: Фото Хенни Гарфанкеля. Бауэри: Фото Керстин Роджерс / Редфернс.Брейнард: Фото Криса Фелвера / Getty Images. Бродки: Андреас Стерзинг. Бучак: любезно предоставлено Джеффри Хендриксом. Бургойн: Фото Патрика МакМуллана / Getty Images. Бертон: Фред В. Макдарра / Getty Images. Каллен: Фотография Роберта Джарда; Авторское право Поместье Роберта Жара. Документы Роберта Джарда: Библиотека Бейнеке в Йельском университете. Каранджи: Фото Гарри Кинга. Чендлер: Ронда Пастер-Корте. Cheren: Новости выплеска / фото Alamy Stock Photo. Чау: вау. Коулман: Том Уоррен. Кори: Фото Хенни Гарфанкеля. Коули: Архивы Майкла Охса.Кокс: Фотография Роберта Джарда; Авторское право Поместье Роберта Жара. Документы Роберта Джарда: Библиотека Бейнеке в Йельском университете. Дэвис, Брэд: © CBS / Предоставлено: Everett Collection. Дэвис, Кейт: Андреас Стерзинг. ДеБарж: © Бобби Холланд / mptvimages.com. ДеСана: Том Уоррен. Донгиа: Фото Хорста П. Хорста / Conde Nast через Getty Images. Дривас: Фото ABC Photo Archives / ABC через Getty Images. Дункан: Автопортрет, Кенн Дункан, c Театр Билли Роуза, Нью-Йоркская публичная библиотека исполнительских видов искусства.Дука: Рон Галелла / WireImage. Иден: Фото Фреда В. МакДарры / Getty Images. Эйхельбергер: Фото Анде Уайленд. Эллис: Фото Тома Гейтса / Архивные фотографии / Getty Images. Eyen: Фото Тима Боксера / Getty Images. Фехер: Фото УИЛЛА РАГОЗЗИНО / Патрика МакМаллана через Getty Images. Ферро: Фотография Роберта Джарда; Авторское право Поместье Роберта Жара. Документы Роберта Джарда: Библиотека Бейнеке в Йельском университете. Фуко: Фото Жана Пьера ФУШЕ / RAPHO / Gamma-Rapho через Getty Images). Фокс: Фотография Роберта Джарда; Авторское право Поместье Роберта Жара.Документы Роберта Джарда: Библиотека Бейнеке в Йельском университете. Франжелла: Андреас Стерзинг. Голтни: Фото Патрика МакМуллана / Getty Images. Горди: фото Джорджа Дюро, любезно предоставлено галереей Артура Роджера. Грейтхаус: Том Уоррен. Хальтсон: Фред В. Макдарра / Getty Images. Харинг: фото Джека Митчелла / Getty Images. Хартман: © Globe Photos / ZUMAPRESS.com. Гибискус: фото Ирен Янг. Худжар: Том Уоррен. Джабара: любезно предоставлено Everett Collection. Джейкобс: Фото: Santi Visalli Inc./Getty Images. Якобсен: Фото Скотта Бромли.Келли: Фредерик Реглейн / Гамма. Керт: Denver Post через Getty Images. Кондолеон: Фотография Роберта Джарда; Авторское право Поместье Роберта Жара. Документы Роберта Джарда: Библиотека Бейнеке в Йельском университете. Ла Турно: Фото Джека Митчелла / Getty Images. Лаво: © Govert De Roos — Mephisto — Christian Rose / Dalle. Леван: Фото Билла Бернстайна. Лопес: любезно предоставлено Нилом Винокуром. Громко: Гэри М. Грин. Мэпплторп: любезно предоставлено Нилом Винокуром. Макгоуэн: Брайан Хагивара. Монтауг: Льютон Коул / Алами. Мур, Фрэнк С.: Предоставлено Нилом Винокуром. Мур, Роберт: Марта Свуп Нью-Йоркская публичная библиотека. Моррисро: Поместье Марка Моррисро (Коллекция Рингье) в Фотомузее Винтертура. Муфарреж: Андреас Стерзинг. Мюллер: Ронда Пастер-Корте. Наварро: Фильм все еще любезно предоставлен ONE National Gay & Lesbian Archives / USC Libraries, Лос-Анджелес. Ниндзя: Адам Фридман / Camera Press / Redux. Номи: Кертис Кнапп / Гетти. Нуреев: Фото Линды Вартугиан / Getty Images. Редд: Жиль Петар. Рене: Фото: Боб Маршак, Copyright Estate Боба Маршака.Райли: Предоставлено Эвереттом. Ритц: Рон Галелла / WireImage / Getty. Ролстон: Фото Мэтью Ролстона. Ротбелл-Миста: Рон Галелла / WireImage / Getty. Рубелл: Соня Московиц / ИЗОБРАЖЕНИЯ / Гетти. Рассел: Фото Джанетт Бекман / любезно предоставлено Audika Records. Святой: Фотография Роберта Джарда; Авторское право Поместье Роберта Жара. Документы Роберта Джарда: Библиотека Бейнеке в Йельском университете. Санчес: Ронда Пастер-Корте. Савар: Андреас Стерзинг. Силз: Рон Галлела LTD / WireImage / Getty. Зайднер: Дэвид Зайднер, [автопортрет Дэвида Зайднера], 1992.© Архив Дэвида Зайднера / Международный центр фотографии (2007.60.90). Пол: Ронда Пастер-Корте. Смит, Джек: Джек Смит Без названия (Студийный портрет), около 1970 года. Черно-белая фотография Сала Терра-Сина. Фото из архива Джека Смита. Предоставлено Gladstone Gallery, Нью-Йорк и Брюссель. Смит, Вилли: Ассошиэйтед Пресс. Снайдер: Андреас Стерзинг. Сен-Жак: Рон Галелла / WireImage / Getty. Стэнтон: Фото любезно предоставлено Larry Stanton Estate. Стирс: Фото любезно предоставлено Хью Стирс Эстейт. Стивенсон: Фото GODLIS.Стюарт: URW / Photoshot / Everett Collection. Саммерс: Изображения для контактной прессы Фрэнка Фурнье. Сильвестр: Энтони Барбоза / Гетти. Тейлор: Фото Рассела Макмастера / любезно предоставлено Michael Taylor Designs. Тек: Пол Тек, ок. 1967. Фотография Эйми Рам, любезно предоставлена ​​Александром и Бонином, Нью-Йорк. Типпет: Фото Грегори Хейслера / любезно предоставлено ABT. Ценг: Том Уоррен. Тубенс: Рон Галелла / WireImage / Getty Vawter: Фотография Кэтрин МакГанн. Вагстафф: Арнольд Ньюман / Гетти. Уорд: Фото Кенна Дункана, c Отделение театра Билли Роуза, Нью-Йоркская публичная библиотека исполнительских видов искусства.Уитмор: Фотография Роберта Джарда; Авторское право Поместье Роберта Жара. Документы Роберта Джарда: Библиотека Бейнеке в Йельском университете. Уилсон: Линн Голдсмит / VCG / Getty. Войнарович: любезно предоставлено Нилом Винокуром. Вулф: Фредерик М. Браун / Стрингер / Гетти. Вонг: Том Уоррен. Йетс Рист: Фотография Роберта Джарда; Авторское право Поместье Роберта Жара. Документы Роберта Джарда: Библиотека Бейнеке в Йельском университете. Зейн: Джек Митчелл / Гетти.

Три десятилетия спустя мужчины, пережившие «чуму геев», говорят

«Если бы вы могли пройти тест, который определил бы, живы ли вы через два года, вы бы его прошли?»

Это был вопрос, Марк С.Кинг сказал, что ему пришлось задаться вопросом в начале 80-х, когда друзья и близкие умирали от «чумы геев».

«Я решил, что хочу знать», — сказал Кинг NBC News, вспоминая день, когда он решил пройти тестирование. 15 марта 1985 года Кингу позвонил его друг, медсестра, которая осторожно проверила его на вирус — он был положительным.

Это было похоже на серию «Сумеречная зона», где все в городе просто начинают исчезать.

Марк С. Кинг

Мужчины-геи не должны были проходить тестирование в то время, сказал Кинг, потому что в любом случае не было единого доступного лечения.Фактически, активисты убеждали людей не принимать это.

«Единственное, что может произойти, — это уволить с работы, выгнать сосед по комнате или отречься от своей семьи», — объяснил Кинг. «Ни один из результатов не был хорош».

«Игра ожидания»

Америка была в панике в тот день, когда Кинг взял трубку телефона в Лос-Анджелесе. По телевидению политики по обе стороны прохода всерьез обсуждали, следует ли помещать геев в карантин.В Белом доме Рейган даже не упомянул СПИД. Но везде и каждый день умирали друзья, родственники, знакомые и партнеры.

«Это было похоже на серию« Сумеречная зона », где все в городе просто начинают исчезать», — сказал Кинг о том времени. «Это был кассир в вашем банке, которого однажды не было. Это был твой любимый бармен. Это парень делал тебе прическу. Они просто перестали там находиться ».

Марк С. Кинг стоит перед знаком Голливуда вскоре после переезда в Калифорнию в 1982 году.Предоставлено Марком С. Кингом

Смерть была последней вещью, с которой Кинг думал, что ему придется столкнуться, когда он переехал в Западный Голливуд из Хьюстона, чтобы продолжить актерскую карьеру. Ему было 24 года, и он хотел наслаждаться жизнью. Вместо этого он оказался в эпицентре вспышки ВИЧ, которая на долгие годы будет определять мир геев в Соединенных Штатах.

«Онемение», — сказал Кинг о том, что он почувствовал, когда его друг сказал ему, что у него положительный результат. «Я просто онемел. Я немедленно отказался от этого. Защитное отрицание.«Может быть, я выживу», — сказал я себе ».

Но у Кинга не было доказательств того, что он это сделает. Все, кого он знал, кому поставили диагноз, попали в больницу и умерли. Бары опустели. По его словам, никто больше не хотел праздновать. Жизнь была, как ее описал Кинг, «игрой на ожидание». И поэтому он ждал того дня, когда у него начнется кашель или появится пятно, и начнется обратный отсчет.

«Я чувствовал, что падают бомбы»

За год до телефонного звонка Кинга, который изменил жизнь Кинга, в 1984 году, Нельсон Вергель поселился в Хьюстоне.Иммигрант из Венесуэлы, Вергель сказал NBC News, что покинул свою родную страну, потому что был геем и искал более благоприятную среду. Будучи честолюбивым инженером-химиком, он также хотел поступить в лучшую школу. Когда он встретил своего парня Кэлвина и влюбился в него на конференции по химическому инжинирингу в Хьюстоне, Вергель решил там жить.

«Чума геев», как ее называли в то время, потрясла нацию. Но считалось, что он ограничен побережьем Сан-Франциско, Лос-Анджелесом и Нью-Йорком.Вергелю было 24 года, он недавно иммигрант, готовый попытаться перебраться в США с новым парнем на руке и многообещающей карьерой. Затем Кальвина проверили за спиной.

Нельсон Вергель (слева) со своим бойфрендом Кальвином (справа), который скончался от СПИДа в 1992 году. Предоставлено Нельсоном Вергелем

«В то время мы не хотели сдавать анализы, потому что мы ничего не могли поделать, и стресс просто убьет тебя быстрее », — сказал Вергель. «Кальвин прошел тестирование без моего ведома и сказал мне, что он положительный.Меня сразу же проверили, и я узнал ».

На фоне депрессии, последовавшей за его диагнозом, друзья Вергеля начали умирать. Он сказал, что «повторно закрылся» как на работе, так и в гей-сообществе. На работе он не хотел, чтобы кто-нибудь узнал, что он гей. В гей-клубах он не хотел, чтобы кто-нибудь узнал, что он ВИЧ-положительный.

Он намеревался приехать в США для новой жизни, но вместо этого он сказал, что чувствовал себя так, будто «приехал в эту страну, чтобы умереть».

Вергель решил занять себя, направив свое беспокойство на волонтерскую работу.Днем он работал в нефтяном бизнесе. Ночью он работал в ВИЧ-клинике, проводя анализы. Он будет посещать больницы, где, по его словам, пациенты с ВИЧ покрываются костюмами HAZMAT. Он был окружен смертью, но альтернатива, по его словам, была хуже. Он думал, что умрет от депрессии, если ничего не сделает.

«Я никогда не был на войне, и когда они сбрасывали бомбы в городе, мне всегда было интересно, как люди могут спать или пережить это. Но просто сделайте это, — сказал он. «Вот как я себя чувствовал.Я чувствовал, что падают бомбы, и в конце концов меня могут поразить бомбы, но мне просто нужно было продолжать ».

«Ярость, ярость и действие»

В 1987 году в Центре общественных услуг для лесбиянок, геев, бисексуалов и трансгендеров на Манхэттене была произнесена пламенная речь. Гей-драматург и активист Ларри Крамер, который впоследствии основал Кризис здоровья геев (GMHC), должен был заменить оратора писательнице и активистке Сьюзан Зонтаг. То, что он сказал в тот день, вызвало движение.

Крамер попросил встать две трети присутствующих и сказал им, что они умрут через пять лет.

«Если моя сегодняшняя речь не напугает вас до чертиков, у нас большие проблемы», — сказал он. «Если то, что вы слышите, не пробуждает в вас гнев, ярость, ярость и действия, у геев не будет будущего здесь, на земле. Сколько времени пройдет, прежде чем вы рассердитесь и начнете сопротивляться? »

Это был призыв к оружию, и Эрик Сойер, молодой гей из северной части штата Нью-Йорк, ответил на него.

У самого Сойера появились симптомы заболевания при раннем проявлении вируса, в 1981 году, когда появились первые сообщения о ВИЧ, а его парень умер от осложнений, вызванных СПИДом, в 1984 году.

«Я был очень зол на то, что не смог ничего сделать, чтобы спасти жизнь моего парня. Я все еще горевал, — сказал Сойер. — Поэтому я обратил свое горе и гнев в ярость и действие и присоединился к движению ».

Сойеру уже сказали «привести свои дела в порядок», когда в 1983 году ему, наконец, поставили диагноз ВИЧ после того, как в течение многих лет проявлялись симптомы. Его врачи сначала сказали ему, что это болезнь Гопкинса или мононуклеоз, но когда источник его недуга был обнаружен, Сойер сказал, что ему сказали, что он умрет через два года.

Мы несли людей, умерших от СПИДа в результате нашего членства, по улицам Нью-Йорка в открытых гробах … Мы говорили, что если вы не собираетесь ничего делать с геноцидом, вам придется переступить через наши тела и запах нашей гниющей плоти.

Эрик Сойер

Ситуация была безрадостной. Сойер сказал, что в Нью-Йорке есть ограниченное количество больниц, которые принимают людей с ВИЧ / СПИДом, и только два похоронных бюро, где хоронят людей, умерших от болезни.Знание своего статуса, сказал он, вторя Кингу и Вергелю, на самом деле вредно, потому что это может означать увольнение или дискриминацию.

Именно в этой тьме зародился рассвет ВИЧ-активизма. В марте 1987 года Крамер и Сойер основали «Коалицию против СПИДа, чтобы высвободить силу» (ACT UP), радикальную организацию, выступающую против СПИДа, которая стремилась пробудить людей к разрушительным последствиям ВИЧ / СПИДа.

Рассвет ВИЧ-активизма

ACT UP начала регулярно проводить встречи и планировать демонстрации гражданского неповиновения.Сойер описал одно из таких действий — попытку вынудить правительство разработать подходящее с медицинской точки зрения жилье для людей с ВИЧ, поскольку в приютах полно людей, больных туберкулезом, к которому люди с ВИЧ особенно восприимчивы.

«Мы собирали мебель, когда квартиры выставляли ее на улице вместе с моим пикапом, и складывали ее в подвале моего дома из коричневого камня в Гарлеме», — сказал Сойер. «Затем мы отвезем мебель на нижний Манхэттен и заблокируем оба конца перекрестка, разместив жилые комнаты, кухни и ванные комнаты на улице и приковав себя цепями к туалетам и кроватям.

Эрик Сойер и его парень Скотт Бернард сидят у Сойера на коленях. Позже Бернард умер от СПИДа. Эрик Сойер

Сойер сказал, что городу пришлось собирать мебель в эвакуаторы и использовать цепные ножницы, чтобы освободить ее от мебели. Нижний Манхэттен был связан и освещался в новостях, а требование ACT UP о приемлемом с медицинской точки зрения жилье, по словам Сойера, было удовлетворено менее чем за месяц.

Среди целей ACT UP был президент Рейган, администрация которого, по словам Сойера, высмеивала людей с ВИЧ.Но другие цели включали Национальный институт здравоохранения и Управление по контролю за продуктами и лекарствами (FDA), поскольку, как выразился Сойер, ничего не предпринимал для разработки лечения. Мэр Нью-Йорка Эд Кох также стал мишенью за то, что ничего не сделал для помощи геям.

В этих целях ACT UP была бескомпромиссной и была готова шокировать людей, которые предпочитали игнорировать эпидемию. Он рассказал о демонстрациях, на которых прах людей, умерших от СПИДа, был рассыпан на лужайке Белого дома.

«Мы несли людей, умерших от СПИДа в результате нашего членства, по улицам Нью-Йорка в открытых гробах в похоронных процессиях, чтобы правительство и общество в целом чувствовали запах, видели и были разрушены нашими буквально мертвыми телами», — сказал Сойер. . «Мы говорили, что если вы не собираетесь ничего делать с геноцидом, вам придется перешагнуть через наши тела и почувствовать запах нашей гниющей плоти».

«Эффект Лазаря»

В 1996 году появились новые лекарственные препараты — комбинация лекарств, известная как антиретровирусная терапия.Для Кинга это была граница между жизнью и смертью и момент, когда он понял, что будет жить.

«Люди, которые умирали, выходили из своих смертных лож», — сказал он. «Только тогда я смог выдохнуть и уйти, ладно, думаю, я еще пробуду здесь».

Связанные

После этого ВИЧ отнял у Кинга многое. Его парень Эмиль умер. Многие из его друзей погибли. Он был готов умереть молодым. Он, как и многие геи и бисексуалы с ВИЧ, держал папку со своими планами поминальной службы.Затем открылся другой мир.

«После этого с нами происходило много всего, — сказал Кинг. «Мы внесли изменения в нашу работу и наши отношения. Я рассталась с давним парнем. Я знаю много мужчин, которые так поступали. Человек, с которым вы хотите умереть, не обязательно тот, с кем вы хотите жить ».

Нельсон ВергельРоберт Соррик

Парень Вергеля, Кальвин, умер в 1992 году от «истощения», как это описывал Вергель. Вергель, поскольку он входит в число мужчин, чей вирус мутировал и был агрессивным, сказал, что всего четыре года назад у него не было неопределяемой вирусной нагрузки.Только тогда, сказал он, он вздохнул с облегчением.

«Вначале вы задаетесь вопросом, как долго продлится это лечение», — сказал Вергель о каждом новом лечении. «Но потом два года назад я подумал, может, это все?»

Сойер, который все еще активен в ACT UP, сказал, что это было в 96-м, когда для него все изменилось.

«Ингибиторы протеазы появились в 96-м, и мое здоровье начало восстанавливаться чудесным образом — эффект Лазаря, о котором люди часто пишут», — сказал он, имея в виду опыт пребывания на пороге смерти, а затем возрождения.

Он также сказал, что примерно в это время фармацевтическим компаниям стало, наконец, выгодно участвовать в исследованиях и производстве препаратов для лечения ВИЧ.

Непрекращающаяся битва

По словам Кинга, Вергеля и Сойера, борьба с ВИЧ / СПИДом далека от завершения. По словам Сойера, существует множество уязвимых сообществ, которые все еще находятся в опасности, и он сказал, что фармацевтические компании по-прежнему ставят прибыль выше благополучия людей с ВИЧ.

«Мы действительно должны работать над социальной справедливостью и справедливыми действиями в отношении здравоохранения и лечения при тестировании на ВИЧ», — сказал он.«Так много людей, бедных и сельских жителей, трансгендеров и цветных людей не имеют равного доступа к тестированию или лечению, и они продолжают умирать от СПИДа».

Марк С. Кинг Джонатан Тиммес

Вергель сказал, что, несмотря на то, что в настоящее время доступны такие усовершенствования в лечении, как доконтактная профилактика (ДКП), которые могут снизить вирусную нагрузку у людей с ВИЧ до неопределяемой, люди с ВИЧ все еще умирают раньше, чем должны, и умственные остаются эмоциональные шрамы.

«У некоторых из нас посттравматический стресс», — сказал он.«Но некоторым из нас удалось отложить это в сторону, потому что жизнь продолжается. Мы празднуем успех антиретровирусных препаратов. Каждый гей должен быть на PrEP, и каждый гей должен пройти тестирование ».

Кинг с тех пор погрузился в мир ВИЧ-активизма, запустив веб-сайт My Fabulous Disease, где он пишет о своем опыте, который начался с телефонного звонка в Западном Голливуде в 1985 году. Он сказал, что хочет, чтобы новое поколение геев гордиться сообществом, из которого они происходят.

«Мы выступили и столкнулись с первобытным страхом и ненавистью в то время, когда никто другой о нас не позаботился», — сказал он.«Мы заботились друг о друге».

ПОДПИСАТЬСЯ НА NBC OUT НА TWITTER , FACEBOOK И INSTAGRAM

AKDS — История цен на акции Arkados Group

Страница истории цен позволяет просматривать историю цен на конец дня. Посетители сайта (не вошедшие на сайт) могут просматривать данные за последние три месяца, в то время как зарегистрированные участники могут просматривать и загружать ежедневные данные за два года для любого диапазона дат, начиная с 1 января 2000 г. Зарегистрированные участники также могут просматривать последние торги и корпоративные действия (У.Только S. обыкновенные акции).

Участники Barchart Premier также могут загружать дополнительные данные с помощью страницы « Исторические данные », где вы можете загружать данные за день, за день, за неделю, за месяц или за квартал. На странице «Исторические данные» доступны

  • Внутридневные данные с шагом в 1 минуту примерно за 10 лет до сегодняшней даты.
  • Ежедневные данные доступны с 01.01.2000.
  • Еженедельные, ежемесячные и квартальные данные доступны с 01.01.1980 (в зависимости от символа).

Вкладка «Ежедневные цены»

Посетители сайта (не авторизованные) видят ежедневную информацию о ценах за 3 месяца для любого символа, который есть на Barchart. При входе на сайт с использованием бесплатного членства или членства Premier вы можете изменить объем отображаемых данных с помощью селектора данных слева от календаря.

Выберите 1 месяц, 3 месяца, 6 месяцев, 1 год или 2 года. При желании вы можете выбрать исторические данные, доступные до двух лет для любого введенного диапазона дат, вплоть до 01.01.2000 (используйте календари в верхней части страницы, чтобы ввести выбранную дату.)

Кроме того, вы можете настроить историю цен для дивидендов (установите флажок «Корректировка дивидендов»). Члены My Barchart и Barchart Premier могут загружать данные в файл .csv для использования в сторонних программах для работы с электронными таблицами.

Вкладка «Последние сделки»

Эта вкладка доступна для участников My Barchart и Barchart Premier. На вкладке «Последние сделки», доступной для акций, фьючерсов и валютных символов США и Канады, отображаются последние 50 сделок по данному символу. Авторизованные пользователи сайта могут загрузить данные в.csv файл.

Условия продажи для символов NYSE / NYSE Arca:

  • @ — Обычная продажа
  • ПУСТО — Условия продажи не требуются в рамках (только для формата длинной сделки)
  • C — Торговля за наличные (клиринг в тот же день)
  • E — Автоматическое исполнение
  • F — Ордер InterMarket Sweep
  • H — Сделка с изменением цены
  • I — Сделка с нечетным лотом
  • K — Правило 127 (только NYSE) или Правило 155 (только Amex)
  • L — Продано последним (поздняя отчетность)
  • M — Официальное закрытие рыночного центра
  • N — Сделка на следующий день (клиринг на следующий день)
  • O — Торговля при открытии рыночного центра
  • P — Предыдущая справочная цена
  • Q — Официальное открытие рыночного центра
  • R — Продавец
  • T — Торговля в продленное время
  • U — Продано продленное время (вне очереди)
  • V — Условная сделка
  • X — Перекрестная сделка
  • Z — Продана (вне очереди)
  • 4 — Установлена ​​производная цена
  • 5 — Сделка повторного открытия в рыночном центре
  • 6 — Сделка закрытия в рыночном центре
  • 7 — Квалифицированная условная сделка
  • 8 — Зарезервирована
  • 9 — Скорректированная консолидированная цена закрытия в соответствии с листинговым рынком

Условия продажи для символов Nasdaq:

  • @ — Обычная продажа
  • A — Приобретение
  • B — Групповая торговля
  • C — Продажа за наличные
  • D — Распределение
  • E — Заполнитель для будущего использования
  • F — InterMarket Sweep
  • G — Групповая продажа
  • 9020 H — Сделка с изменением цены
  • I — Сделка с нечетным лотом
  • K — Сделка по правилу 155 (NYSE Arca)
  • L — Последняя продажа
  • M — Официальное закрытие рыночного центра
  • N — На следующий день
  • O — Начальные распечатки
  • P — Предварительная справочная цена
  • Q — Официальное открытие торгового центра
  • R — Продавец
  • S — Сплит-торговля
  • T — Форма T 9020 4
  • U — Продленные торговые часы (продажа вне очереди)
  • V — Сделка с акциями и опционами
  • W — Сделка по средней цене
  • X — Перекрестная торговля
  • Y — Обычная сделка с желтым флагом
  • Z — Продана (вне очереди) Последовательность)
  • 1 — Остановленные акции (Обычная сделка)
  • 4 — С производной ценой
  • 5 — Повторно открывающиеся отпечатки
  • 6 — Заключительные отпечатки
  • 8 — Заполнитель для 611 Exempt
  • 9 — Исправленное консолидированное закрытие (для каждого листинга market)

Вкладка «Корпоративные действия»

Эта вкладка доступна для участников My Barchart и Barchart Premier.Вкладка Действия корпорации показывает дробление акций, дивиденды и прибыль. Корпоративные действия доступны только для акций США и Канады. Кроме того, участники My Barchart видят данные за последние два года, а участники Barchart Premier будут видеть корпоративные действия, начиная с 1 января 2000 года.

Обновления данных

Для страниц, показывающих просмотры в течение дня, мы используем данные текущего сеанса с новые данные о ценах появятся на странице, обозначенной «вспышкой». Акции: задержка 15 минут (данные Cboe BZX для U.S. Equities в режиме реального времени), ET. Объем отражает консолидированные рынки. Фьючерсы и Forex: задержка 10 или 15 минут, CT.

Список символов , представленных на странице, обновляется каждые 10 минут в течение торгового дня . Тем не менее, новые акции не добавляются автоматически и не меняются рейтинги на странице до тех пор, пока сайт не выполнит 10-минутное обновление.

Сортировка страниц

Страницы изначально сортируются в определенном порядке (в зависимости от представленных данных). Вы можете пересортировать страницу, щелкнув заголовок любого столбца в таблице.

Представления

Большинство таблиц данных можно анализировать с помощью «Представлений». Представление просто представляет символы на странице с другим набором столбцов. Участники сайта также могут отображать страницу, используя Custom Views . (Просто создайте бесплатную учетную запись, войдите в систему, затем создайте и сохраните пользовательские представления для использования в любой таблице данных.)

В каждом представлении есть столбец «Ссылки» в дальнем правом углу для доступа к обзору котировок символа, диаграмме, котировкам опций (если доступно), страницу с мнением в виде гистограммы и технический анализ.Стандартные представления, которые можно найти по всему сайту, включают:

  • Основной вид : символ, имя, последняя цена, изменение, процентное изменение, максимум, минимум, объем и время последней сделки.
  • Технический взгляд : символ, имя, последняя цена, сегодняшнее мнение, 20-дневная относительная сила, 20-дневная историческая волатильность, 20-дневный средний объем, 52-недельный максимум и 52-недельный минимум.
  • Просмотр производительности : символ, имя, последняя цена, взвешенная альфа, изменение в процентах с начала года, изменение в процентах за 1, 3 и 1 год.
  • Фундаментальный обзор : Доступно только на страницах акций, показывает символ, название, взвешенную альфа-версию, рыночную капитализацию, коэффициент P / E. Прибыль на акцию, бета, рентабельность капитала и цена / продажи
Таблица данных Развернуть

Таблицы данных, уникальные для Barchart.com, содержат параметр «развернуть». Щелкните значок «+» в первом столбце (слева), чтобы «развернуть» таблицу для выбранного символа. Прокрутите виджеты с различным содержимым, доступным для символа. Щелкните любой из виджетов, чтобы перейти на полную страницу.

Горизонтальная прокрутка широких таблиц

Особенно при использовании настраиваемого представления вы можете обнаружить, что количество выбранных столбцов превышает доступное пространство для отображения всех данных. В этом случае таблицу необходимо прокручивать по горизонтали (слева направо), чтобы просмотреть всю информацию. Для этого вы можете либо прокрутить таблицу до конца и использовать полосу прокрутки таблицы, либо прокрутить таблицу с помощью встроенной прокрутки браузера:

  • Щелкните левой кнопкой мыши в любом месте таблицы.
  • Используйте на клавиатуре стрелки влево и вправо для прокрутки таблицы.
  • Повторите это в любом месте при перемещении по таблице, чтобы включить горизонтальную прокрутку.
FlipCharts

Флипчарты, также уникальные для гистограмм, позволяют прокручивать все символы в таблице в режиме просмотра диаграммы. При просмотре FlipCharts вы можете применить собственный шаблон диаграммы, дополнительно настроив способ анализа символов. FlipCharts — это бесплатный инструмент, доступный участникам сайта.

Загрузить

Загрузить — это бесплатный инструмент, доступный участникам сайта. Этот инструмент загрузит файл .csv для отображаемого представления. Для динамически генерируемых таблиц (таких как Stock или ETF Screener), где вы видите более 1000 строк данных, загрузка будет ограничена только первыми 1000 записями в таблице. Для других статических страниц (таких как список компонентов Russell 3000) будут загружены все строки.

Бесплатные участники ограничены до 5 загрузок в день, в то время как участники Barchart Premier могут загружать до 100.csv файлов в день.

Примечание : из-за лицензионных ограничений основные данные Канады нельзя загрузить с Barchart.com. В этом случае вы увидите «N / L» в столбце загруженных файлов.

Если вам требуется более 100 загрузок в день, свяжитесь с отделом продаж Barchart по телефону 866-333-7587 или напишите по адресу [email protected] для получения дополнительной информации или дополнительных опций по историческим рыночным данным.

Новые максимумы

Этот виджет показывает, сколько раз этот символ достигал нового максимума за определенные периоды, от последних 5 дней до последних 20 лет.

Для каждого периода столбец «Процент от последнего» показывает, где текущая цена соотносится с максимальной ценой за этот период. В случае отрицательного значения текущая цена намного ниже, чем самая высокая цена, зарегистрированная за период. При положительном значении текущая цена намного выше, чем самая высокая цена за этот период.

Новые минимумы

Этот виджет показывает, сколько раз этот символ достиг новой минимальной цены за определенные периоды, от последних 5 дней до последних 20 лет.

Для каждого периода столбец «Процент от последнего» показывает, где текущая цена соотносится с минимальной ценой за этот период. При положительном значении текущая цена намного выше самой низкой цены за этот период. В случае отрицательного значения текущая цена намного ниже самой низкой цены за период.

Байер продолжал продавать лекарство, способствовавшее распространению ВИЧ

Спустя почти 20 лет точное количество человеческих жертв этих маркетинговых решений трудно, если не невозможно, задокументировать.

Но только в Гонконге и на Тайване более 100 больных гемофилией заразились ВИЧ после использования старого лекарства Каттера, как показывают записи и интервью. Многие с тех пор умерли. Cutter также продолжал продавать старые продукты после февраля 1984 года в Малайзии, Сингапуре, Индонезии, Японии и Аргентине, как показывают отчеты.

«Это самые компрометирующие внутренние документы фармацевтической отрасли, которые я когда-либо видел», — сказал д-р Сидни Вулф, который в качестве директора Группы исследований общественного здоровья граждан изучал отраслевую практику в течение 30 лет.

В заявлении Bayer говорится, что Cutter «вел себя ответственно, этично и гуманно», продавая старый продукт за границу. Каттер продолжал продавать старое лекарство, заявило в нем, потому что некоторые покупатели сомневались в эффективности нового лекарства, а также потому, что некоторые страны не спешили одобрять его продажу.

«Решения, принятые почти два десятилетия назад, были основаны на лучшей научной информации того времени и соответствовали действующим правилам», — говорится в заявлении.

Лекарство, называемое концентратом фактора VIII, по существу обеспечивает недостающий ингредиент, без которого кровь больных гемофилией не может свернуться.Делая себе инъекции, больные гемофилией могут остановить кровотечение или предотвратить кровотечение; некоторые используют его до трех раз в неделю. Это помогло больным гемофилией вести нормальную жизнь.

Но в первые годы эпидемии СПИДа он стал убийцей. В медицине использовались пулы плазмы от 10 000 или более доноров, и, поскольку до сих пор не проводилось скринингового теста на вирус СПИДа, риск передачи болезни был высоким; даже небольшое количество ВИЧ-положительных доноров может заразить весь пул.

В США СПИД передался тысячам больных гемофилией, многие из которых умерли в результате одной из самых страшных медицинских катастроф, связанных с наркотиками в истории. Не признавая никаких нарушений, Bayer и три другие компании, которые производили концентрат, выплатили больным гемофилией около 600 миллионов долларов (900 миллионов долларов) для урегулирования более чем 15-летних судебных исков.

The New York Times

Самые просматриваемые в мире

Загрузка

A.k.d. в Южной Голландии, IL

ПРЕДУПРЕЖДЕНИЕ: Всего в нашей базе данных 2 займа ГЧП для предприятий с названием «A.k.d.s »в Южной Голландии, штат Иллинойс. Причина получения нескольких кредитов может включать в себя одноименные, но не связанные предприятия, несколько филиалов одного и того же бизнеса, ошибочные несколько заявок или возможное мошенничество. См. список кредитов под этим именем и дополнительную информацию ниже.

A.k.d.s — это компания с ограниченной ответственностью (LLC), расположенная в Южной Голландии, штат Иллинойс, которая получила от SBA ссуду в связи с коронавирусом в размере 20 833,00 долларов США в феврале 2021 года.

Компания заявила о себе как о бизнесе, принадлежащем чернокожим или афроамериканским мужчинам, и наняла как минимум одного человека в течение соответствующего периода ссуды.

$ Информация о займе ГЧП

A.k.d.s получили заем на выплату зарплаты в размере 20 833 долларов через Itria Ventures LLC, который был одобрен в феврале 2021 года.


Расчет ГЧП Примечание: Общая сумма ссуды ГЧП, которую может получить правомочное предприятие или физическое лицо, основывается на 2,5-кратном размере их среднемесячных расходов на заработную плату в 2019 году , ограниченных 100000 долларов США в год на одного сотрудника.

Зарегистрированное использование процесса ГЧП:

Компания сообщила об использовании средств по займу ГЧП на следующие расходы:

Заработная плата: 20 831 $ — Коммунальные услуги: 1 $ — Проценты по ипотеке: $ 0 — Аренда: $ 0 — Рефинансирование EIDL: $ 0 — Здравоохранение: $ 0 — Проценты по долгу: $ 0

Номер займа SBA 4314678407

Деловая информация — А.k.d.s в Южной Голландии, IL

Похожие компании рядом с South Holland

В районе Южной Голландии 94 предприятия отрасли «Общие грузоперевозки, местные» получили ссуду в рамках ГЧП. Эти местные предприятия сообщили о среднем 2 сотрудниках (по сравнению с 1 этой компанией) и получили среднюю ссуду ГЧП в размере 23 389 долларов (по сравнению с 20 833 долларами этой компании) .

Похожие компании поблизости, получившие финансирование в рамках ГЧП:

Ашиа Лонг
Роббинс, Иллинойс

Заем на сумму 20 000 долларов США по ГЧП

Akeem Oluleye
South Holland, IL

Заем на сумму 17 180 долларов США по ГЧП

Валенсия Хемфилл
Южная Голландия, IL

$ 20 832 заем ГЧП

Джесси Дэвис
Южная Голландия, IL

Заем на сумму 14 500 долларов США по ГЧП

Jamon Render
Южная Голландия, IL

$ 20,332 заем ГЧП

Десмонд Уолтон
Южная Голландия, IL

$ 20 833 заем ГЧП

Тимоти Таунсел
Южная Голландия, IL

$ 12 037 заем ГЧП

Intermodal Solutions Corp
Южная Голландия, IL

$ 57 650 заем ГЧП

Рейнард Берд
Южная Голландия, Иллинойс

$ 20 832 заем ГЧП

Статистика сравнения отраслевых ППС

По всей стране 121 560 предприятий отрасли «Общие грузовые автоперевозки, местные» получили в общей сложности 5 541 451 028 долларов.00 в кредитах ГЧП. На эти предприятия приходится 1% от общего числа поданных заявок на ГЧП, и они получили 1% от общего объема выделенного финансирования в рамках ГЧП.

Получатели ГЧП в этой отрасли сообщают, что в среднем сотрудников 5 , На 400% выше, чем на , чем у A.k.d.s зарегистрировано 1 сотрудник, и получено в среднем ссуды ГЧП в размере 45 586 долларов, что на 119% больше, чем на , чем ссуда этой компании в размере 20 833 долларов.

A.k.d.s — это компания с ограниченной ответственностью (LLC), расположенная в Южной Голландии, штат Иллинойс, которая получила от SBA ссуду для ГЧП, связанную с коронавирусом, в размере 15 750 долларов США.00 в марте 2021 г.

Компания заявила о себе как о бизнесе, принадлежащем чернокожим или афроамериканским мужчинам, и наняла как минимум одного человека в течение соответствующего периода ссуды.

$ Информация о займе ГЧП

A.k.d.s получили заем на выплату зарплаты в размере 15 750 долларов США через Itria Ventures LLC, который был одобрен в марте 2021 года.


Расчет ГЧП Примечание: Общая сумма кредита ГЧП, которую может получить правомочное предприятие или физическое лицо, основывается на 2.В 5 раз больше их среднемесячных расходов на заработную плату в 2019 году, равных , максимум 100000 долларов США в год на одного сотрудника.

Зарегистрированное использование процесса ГЧП:

Компания сообщила об использовании средств по займу ГЧП на следующие расходы:

Заработная плата: 15746 долларов — Коммунальные услуги: 1 $ — Проценты по ипотеке: $ 0 — Аренда: $ 0 — Рефинансирование EIDL: $ 0 — Здравоохранение: $ 0 — Проценты по долгу: $ 0

Номер займа SBA 3529208703

Деловая информация — A.k.d.s в Южной Голландии, IL

Похожие компании рядом с South Holland

В районе Южной Голландии 94 предприятия отрасли «Общие грузоперевозки, местные» получили ссуду в рамках ГЧП.Эти местные предприятия сообщили в среднем о 2 сотрудниках (по сравнению с 1 этой компанией) и получили среднюю ссуду ГЧП в размере 23389 долларов (по сравнению с 15750 долларов США) этой компании.

Похожие компании поблизости, получившие финансирование в рамках ГЧП:

Ашиа Лонг
Роббинс, Иллинойс

Заем на сумму 20 000 долларов США по ГЧП

Akeem Oluleye
South Holland, IL

Заем на сумму 17 180 долларов США по ГЧП

Валенсия Хемфилл
Южная Голландия, IL

$ 20 832 заем ГЧП

Джесси Дэвис
Южная Голландия, IL

Заем на сумму 14 500 долларов США по ГЧП

Jamon Render
Южная Голландия, IL

$ 20,332 заем ГЧП

Десмонд Уолтон
Южная Голландия, IL

$ 20 833 заем ГЧП

Тимоти Таунсел
Южная Голландия, IL

$ 12 037 заем ГЧП

Intermodal Solutions Corp
Южная Голландия, IL

$ 57 650 заем ГЧП

Рейнард Берд
Южная Голландия, Иллинойс

$ 20 832 заем ГЧП

Статистика сравнения отраслевых ППС

По всей стране 121 560 предприятий отрасли «Общие грузовые автоперевозки, местные» получили в общей сложности 5 541 451 028 долларов.00 в кредитах ГЧП. На эти предприятия приходится 1% от общего числа поданных заявок на ГЧП, и они получили 1% от общего объема выделенного финансирования в рамках ГЧП.

Получатели ГЧП в этой отрасли сообщают, что в среднем сотрудников 5 , На 400% выше, чем на , чем у A.k.d.s зарегистрировано 1 сотрудник, и получено в среднем ссуды ГЧП в размере 45 586 долларов США, что на 189% выше, чем на , чем ссуда этой компании в размере 15 750 долларов США.

Информационная политика FederalPay в области ГЧП
Администрация малого бизнеса (SBA) опубликовала данные о ссуде

с защитой зарплаты для всех частных компаний, получивших ссуду в рамках ГЧП.

Вся информация, отображаемая на этой странице, является общедоступной в соответствии с рекомендациями по займам в рамках ГЧП в соответствии с 5 U.S.C. § 552 (Закон о свободе информации) и 5 ​​U.S.C. § 552a (Закон о конфиденциальности) и публикуется без изменений, как это предусмотрено SBA. FederalPay не изменяет данные и не претендует на их точность.

Любые исправления или модификации этих данных могут быть сделаны только через SBA. Для получения дополнительной информации см. Политику в отношении данных FederalPay PPP.

1. Расчет заработной платы и оценки заработной платы предполагают, что заемщик использовал стандартный расчет ППС 2,5 х среднемесячные расходы на заработную плату за 2019 год для определения права на получение кредита ГЧП. Методы расчета зависят от типа объекта. Пожалуйста, ознакомьтесь с последними официальными правилами расчета ГЧП SBA для полного объяснения методов расчета суммы кредита ГЧП.

2. Если заявленное количество сотрудников компании, разделенное на максимальную сумму диапазона ППС в соответствии с SBA, превышает 100 000 долларов, расчетный максимальный размер кредита, полученного компанией, может быть скорректирован, чтобы предположить, что годовая зарплата на одного сотрудника не превышала 100 000 долларов. используется в приложении PPP.Хотя сотрудники компании могут зарабатывать больше, 100 тысяч долларов на сотрудника — это максимальная сумма, которую можно использовать при расчетах права на участие в ГЧП.

Были ли полезны инструменты открытых данных FederalPay.org? Рассмотрите возможность пожертвования!

{\ rtf1 \ ansi \ ansicpg1252 \ deff0 \ nouicompat \ deflang1033 \ deflangfe2052 {\ fonttbl {\ f0 \ fswiss \ fprq2 \ fcharset0 Arial;} {\ f1 \ fnil \ fcharset0 Arial0nil; Times New Roman;} {\ f3 \ froman \ fprq2 \ fcharset0 Times New Roman;} {\ f4 \ fnil Times New Roman;} {\ f5 \ fnil Arial;} {\ f6 \ fnil \ fcharset1 Символ пользовательского интерфейса Segoe;} { \ f7 \ fnil \ fcharset2 Symbol;}} {\ colortbl; \ red0 \ green0 \ blue0; \ red255 \ green255 \ blue0; \ red0 \ green0 \ blue255; \ red0 \ green255 \ blue255; \ red255 \ green0 \ blue255; \ red17 \ green85 \ blue204; \ red255 \ green255 \ blue255; \ red42 \ green42 \ blue42; \ red23 \ green54 \ blue93;} {\ * \ generator Riched20 10.0.19041} {\ * \ mmathPr \ mnaryLim0 \ mdispDef1 \ mwrapIndent1440} \ viewkind4 \ uc1 \ pard \ nowidctlpar \ widctlpar \ sl240 \ slmult1 \ qc \ cf1 \ b \ f0 \ fs28 \ par \ номинал Национальная ассоциация собачьего запаха, LLC \ super \ ‘ae \ par \ nosupersub \ fs32 \ par \ fs28 Официальный NACSW \ ’99 \ par \ b0 \ номинал \ b ИСПЫТАНИЯ НА ОПРЕДЕЛЕНИЕ ЗАПАХОВ \ par \ b0 \ fs24 \ par \ b Запахи протестированы: \ cf0 \ b0 \ f1 \ fs22 \ lang9 Береза, анис, гвоздика \ cf1 \ f0 \ lang1033 \ par \ fs24 \ par \ b Дата: \ cf0 \ b0 \ f1 \ fs22 \ lang9 Воскресенье, 4 апреля 2021 г. \ cf1 \ highlight2 \ f0 \ lang1033 \ par \ highlight0 \ fs24 \ par \ pard \ widctlpar \ qc \ b Место проведения теста: \ cf0 \ f1 \ lang9 Alaska Dog Sports \ par \ b0 510 W 53 \ super rd \ nosupersub Ave (выход с W54th) \ line Анкоридж, Аляска 99518 \ par \ pard \ nowidctlpar \ widctlpar \ sl240 \ slmult1 \ qc \ cf1 \ highlight2 \ f0 \ fs22 \ lang1033 \ par \ highlight0 \ fs24 \ par \ b Тестовый хост: \ cf0 \ b0 \ f1 \ lang9 Alaska Dog Sports, LLC \ cf1 \ f0 \ fs20 \ lang1033 \ par \ fs24 \ par \ b Секретарь тестирования: \ cf0 \ b0 \ f1 \ lang9 Пит или Лиза Саммерс \ line POB 232521 Анкоридж AK, 99523 \ line {\ cf3 \ ul {\ field {\ * \ fldinst {HYPERLINK «mailto: [email protected] «}} {\ fldrslt{[email protected]}}}} \ cf1 \ f0 \ fs22 \ lang1033 \ par \ pard \ nowidctlpar \ widctlpar \ sl240 \ slmult1 \ highlight4 \ i \ fs20 \ par \ pard \ nowidctlpar \ widctlpar \ sl240 \ slmult1 \ qc \ highlight5 \ par \ pard \ widctlpar \ qc \ highlight0 \ b \ i0 \ fs24 Официальное лицо, выдающее сертификат: \ cf0 \ b0 \ f1 \ lang9 Ян Эрвин \ par \ pard \ nowidctlpar \ widctlpar \ sl240 \ slmult1 \ qc \ cf1 \ f0 \ lang1033 \ par \ highlight5 \ par \ pard \ widctlpar \ qc \ highlight0 \ b Судья: \ cf0 \ b0 \ f1 \ lang9 Деб Гиллис \ par \ pard \ nowidctlpar \ widctlpar \ sl240 \ slmult1 \ qc \ cf1 \ f0 \ lang1033 \ par \ номинал \ pard \ widctlpar \ qc \ b Стоимость входа: \ cf0 \ b0 \ f2 \ lang9 $ 35.00 долларов США за первый запах и 15 долларов США за каждый дополнительный запах за каждую собаку \ номинальной стоимости. \ pard \ nowidctlpar \ widctlpar \ sl240 \ slmult1 \ qc \ cf1 \ f0 \ fs22 \ lang1033 \ par \ fs24 \ par \ pard \ widctlpar \ qc \ b Количество записей: \ cf0 \ b0 \ f1 \ fs22 \ lang9 Ограничено 60 тестами \ par \ pard \ nowidctlpar \ widctlpar \ sl240 \ slmult1 \ qc \ cf1 \ f0 \ lang1033 \ par \ b \ fs24 \ par \ b0 \ номинал \ pard \ widctlpar \ sl240 \ slmult1 \ fs22 NACSW требует, чтобы все участники придерживались социального дистанцирования — поддерживая дистанцию ​​6 \ r — на всех испытаниях до дальнейшего уведомления, даже если ваша юрисдикция смягчила свои требования или рекомендации.Требования социального дистанцирования на мероприятии ОБЯЗАТЕЛЬНЫ. \ I нет возможности \ i0 взаимно согласиться быть ближе друг к другу, чем 6 \ rquote. Пожалуйста, учитывайте, что ваша близость или просьба о близости могут причинить другим неудобство. Просмотрите документ {\ f3 \ fs24 {\ field {\ * \ fldinst {HYPERLINK «https://drive.google.com/open?id=1pMMICXeMDJV2iFxAFOuPAMXEBoRmaik0qzSE4tiZuSI»}\fldrslt{\f\cful3 \ f0 \ fs22 COVID}}}} {\ f3 \ fs24 {\ field {\ * \ fldinst {HYPERLINK «https://drive.google.com/open?id=1pMMICXeMDJV2iFxAFOuPAMXEBoRmaik0qzSE4tiZuSI»}} {\ fldinst cf3 \ cf6 \ ul \ f0 \ fs22 -19}}}} {\ cf1 \ ulnone \ f3 \ fs24 {\ field {\ * \ fldinst {HYPERLINK «https: // диск.google.com/open?id=1pMMICXeMDJV2iFxAFOuPAMXEBoRmaik0qzSE4tiZuSI»}}{\fldrslt{\ul\cf3\cf6\ul\f0\fs22 Event}}}} {\ cf1 \ ulnone \ f3 \ fs24 {ГИПЕРССЫЛКА «https://drive.google.com/open?id=1pMMICXeMDJV2iFxAFOuPAMXEBoRmaik0qzSE4tiZuSI»}{\fldrslt{\ul\cf3\cf6\ul\f0\fs22}}}} {\ cf1 \ fs22}}}} {\ field {\ * \ fldinst {HYPERLINK «https://drive.google.com/open?id=1pMMICXeMDJV2iFxAFOuPAMXEBoRmaik0qzSE4tiZuSI»}}{\fldrslt{\ul\cf3\cf6\ul\f0 }\fs} {\ cf1 \ ulnone \ f3 \ fs24 {\ field {\ * \ fldinst {ГИПЕРССЫЛКА «https: // drive.google.com/open?id=1pMMICXeMDJV2iFxAFOuPAMXEBoRmaik0qzSE4tiZuSI»}}{\fldrslt{\ul\cf3\cf6\ul\f0\fs22}}}} \ cf1 \ ulnone \ f0 \ fs22s, расположенный в каталоге Google docs22, членство на 08.05.2020. \ par \ номинал В дополнение к этим рекомендациям в этом испытании в связи с COVID-19 действуют следующие изменения и рекомендации. \ Par \ pard \ widctlpar \ sb240 \ sl240 \ slmult1 Все участники (участники и волонтеры) должны будут приносить и носить маски для лица, когда они не в автомобиле.\ номинал \ line Участников просят оставаться в своих транспортных средствах как можно дольше. Разрешена эксплуатация транспортных средств. \ Par \ line Участников просят привезти с собой дезинфицирующее средство для рук и дезинфицирующие салфетки. \ par \ pard \ widctlpar \ sb240 \ sa240 \ sl240 \ slmult1 Если есть какие-либо изменения или обновления, эта информация будет включена в вашу окончательную информацию, которая будет отправлена ​​по электронной почте за 1-2 недели до выходных ОРТ. \ номинал \ pard \ nowidctlpar \ widctlpar \ sl240 \ slmult1 \ fs24 \ par \ trowd \ trgaph208 \ trleft-158 \ trbrdrl \ brdrs \ brdrw10 \ brdrbtw \ brdrbar \ trbrdrt \ brdrs \ brdrw10 \ trbrdrr \ brdrs \ brdrw10 \ trbrdrb \ brdrs \ brdrw103 \ trpaddlrb \ brdrs \ brdrw103 \ trpaddrw108 \ trpaddlrw103 \ trpaddlr \ clvertalc \ clcfpat9 \ clcbpat9 \ clshdng10000 \ clbrdrl \ brdrw20 \ brdrs \ brdrcf1 \ clbrdrt \ brdrw20 \ brdrs \ brdrcf1 \ clbrdrr \ brdrw20 \ brdrs \ brdrcf1 \ brdrdrs \ brdrdrs \ brdr7220 \ brdrdrs \ brdr7220 \ pard \ intbl \ nowidctlpar \ sl240 \ slmult1 \ cf7 ВХОДНАЯ ИНФОРМАЦИЯ В ОРТ \ ячейка \ строка \ pard \ nowidctlpar \ sl240 \ slmult1 \ cf1 \ ul \ b \ fs22 \ par \ pard \ qc \ ulnone \ fs24 Дата открытия: \ cf0 \ b0 \ f1 \ fs22 \ lang9 Четверг, 25 февраля 2021 г. \ par \ pard \ nowidctlpar \ sl240 \ slmult1 \ cf1 \ f0 \ lang1033 at \ cf0 \ f1 \ lang9 08:00 Стандартное время Аляски \ номинал \ cf1 \ f0 \ fs20 \ lang1033 \ par \ b \ fs24 Закрытие записей: \ cf0 \ b0 \ f1 \ lang9, понедельник, 22 марта 2021 г. \ cf1 \ f0 \ fs22 \ lang1033 в \ cf0 \ f1 \ fs24 \ lang9 2100 (21:00) по стандартному времени Аляски или когда ограничения на вход соответствует \ line \ cf1 \ f0 \ fs22 \ lang1033 \ par \ номинал \ b \ fs24 Политика в отношении второй собаки: \ cf0 \ b0 \ f1 \ fs22 \ lang9 Каждый проводник может тестировать 2 собак на каждый запах \ cf1 \ highlight2 \ f0 \ lang1033 \ par \ highlight4 \ par \ highlight0 \ ul \ b Метод ввода \ par \ pard \ nowidctlpar \ cf0 \ ulnone \ b0 \ f2 \ fs24 \ lang9 Сначала получено при онлайн-регистрации или по почте USPS.Любая ночь \ номинал Для доставки \ pard необходимо установить флажок «Отказ от подписи». \ par \ f1 \ par \ pard \ nowidctlpar \ cf1 \ b \ fs22 Записи \ par \ pard \ nowidctlpar \ b0 \ fs20 ТЕЛЕФОН, ФАКС, НЕПОДПИСАННЫЕ ЗАПИСИ И ЗАПИСИ, ПОЛУЧЕННЫЕ БЕЗ КОМИССИИ, не принимаются. \ par ВОЗВРАЩЕННЫЕ ЧЕКИ не являются действительным вступительным взносом. \ ~ (За возвращенные чеки взимается комиссия NSF в размере 25 долларов США). \ Par ЗАПИСЬ ДОЛЖНА БЫТЬ СДЕЛАНА НА ПРИЛОЖЕННОЙ ФОРМЕ ЗАЯВКИ.\ номинал ЗАПИСЬ ЗАПРЕЩАЕТСЯ, И ЗАЯВКА НЕ ПРИНИМАЕТСЯ, КОТОРАЯ УКАЗЫВАЕТ ЛЮБЫЕ УСЛОВИЯ ЕГО ПРИНЯТИЯ. \ ~ \ ~ \ Par ВЛАДЕЛЬЦА несут ответственность за ошибки при заполнении регистрационных форм. \ Par Записи EXPRESS MAIL должны включать подпись, позволяющую оставить их по адресу секретаря \ rquote без подписи получателя \ rquote. \ Par \ pard \ nowidctlpar \ sl240 \ slmult1 \ f0 \ fs22 \ lang1033 \ par \ highlight7 \ b Уведомление \ par \ pard \ cf0 \ highlight0 \ b0 \ f1 \ lang9 Подтверждение принятия / подтверждения записи, \ cf8 будет отправлено по электронной почте при обработке записей \ cf0 для тех обработчиков, которые предоставляют адрес электронной почты.Если вам требуется подтверждение через USPS, вы должны предоставить конверт с адресом и печатью с вашей записью. \ Cf8 Квитанция об оплате и уведомление являются подтверждением вашей заполненной записи. \ Cf0 \ par \ pard \ nowidctlpar \ sl240 \ slmult1 \ cf1 \ ul \ b \ f0 \ lang1033 \ par Требования к поступающим \ номинал \ ulnone \ b0 \ par \ b Квалифицированная заявка соответствует \ ul всем \ ulnone следующим критериям: \ par \ pard {\ pntext \ f7 \ ‘B7 \ tab} {\ * \ pn \ pnlvlblt \ pnf7 \ pnindent360 {\ pntxtb \’ B7}} \ fi-359 \ li720 \ cf0 \ b0 \ f1 \ lang9 Завершите запись во время указанные временные рамки входа.\ номинал {\ pntext \ f7 \ ‘B7 \ tab} Обработчик является членом NACSW \ ’99 на 2020-2021 год членства (1 августа — 31 июля 21). \ номинал {\ pntext \ f7 \ ‘B7 \ tab} Собака зарегистрирована в NACSW \ ’99. \ par {\ pntext \ f7 \ ‘B7 \ tab} Собаки, участвующие в тесте на распознавание запахов (ORT), должны быть не моложе шести месяцев. \ par \ пар \ номинал Для получения информации о том, как зарегистрироваться или стать участником, перейдите на страницу {\ cf3 {\ field {\ * \ fldinst {HYPERLINK «http://www.nacsw.net/»}{\fldrslt{\ul\cf3\ul}. }}} {\ f1 \ fs22 {\ field {\ * \ fldinst {ГИПЕРССЫЛКА www.NACSW.net}} {\ fldrslt {www.NACSW.net \ ul0 \ cf0}}}} \ cf6 \ ulnone \ f1 \ fs22 \ par \ pard \ nowidctlpar \ sl240 \ slmult1 \ ul \ f0 \ lang1033 \ par \ cf1 \ ulnone \ b Право на участие в судебном разбирательстве и ОРЦ \ par \ b0 Чтобы получить право на участие в первом периоде розыгрыша, необходимо пройти ОРТ не менее чем за 14 дней до даты открытия. \ line \ line \ par \ ul \ b Основное описание теста \ b0 \ par \ ulnone Чтобы успешно пройти ОРТ, собака должна определить местоположение целевого запаха, а проводник должен правильно вызвать \ ldblquote alert \ rdblquote в течение трех минут.Тест на распознавание запахов состоит из упражнения, которое требует 12 закрытых ящиков, расположенных в виде \ i одного или \ i0 двух рядов, расположенных на расстоянии не менее 48 \ rdblquote друг от друга. ОРТ проводится на поводке. Полную информацию можно получить на сайте NACSW. \ Par \ номинал \ i Все экспоненты должны получить и просмотреть копию \ b NACSW K9 NOSE WORK \ ‘ae ПРАВИЛА. \ b0 \ i0 Вы можете получить его на их веб-сайте: {\ f3 \ fs24 {\ field {\ * \ fldinst {HYPERLINK «http://www.nacsw.net»}} {\ fldrslt {\ ul \ cf3 \ f0 \ fs22}}}} {\ f3 \ fs24 {\ field {\ * \ fldinst {ГИПЕРССЫЛКА «http: // www.nacsw.net «}} {\ fldrslt {\ ul \ cf3 \ cf6 \ ul \ f0 \ fs22 www}}}} {\ f3 \ fs24 {\ field {\ * \ fldinst {ГИПЕРССЫЛКА» http: //www.nacsw .net «}} {\ fldrslt {\ ul \ cf3 \ cf6 \ ul \ f0 \ fs22.}}}} {\ cf1 \ ulnone \ f3 \ fs24 {\ field {\ * \ fldinst {ГИПЕРССЫЛКА» http: // www.nacsw.net «}} {\ fldrslt {\ ul \ cf3 \ cf6 \ ul \ f0 \ fs22 NACSW}}}} {\ cf1 \ ulnone \ f3 \ fs24 {\ field {\ * \ fldinst {ГИПЕРССЫЛКА» http : //www.nacsw.net «}} {\ fldrslt {\ ul \ cf3 \ cf6 \ ul \ f0 \ fs22.}}}} {\ cf1 \ ulnone \ f3 \ fs24 {\ field {\ * \ fldinst { ГИПЕРССЫЛКА «http://www.nacsw.net»}} {\ fldrslt {\ ul \ cf3 \ cf6 \ ul \ f0 \ fs22 net}}}} \ cf1 \ ulnone \ f0 \ fs22 \ par \ pard \ nowidctlpar \ sb240 \ sa120 \ sl240 \ slmult1 \ b Ни одной собаке не будет предоставлено более одного места в одном и том же тесте запаха на одном мероприятии.\ line \ par \ pard \ sl240 \ slmult1 \ ul Результаты \ b0 \ par \ ulnone Во время восстановления запаха официальный лист результатов выносится на стоянку, где тестировщики могут сфотографировать или просмотреть. Мы рекомендуем вам проверить свои результаты в официальном листе результатов и сообщить своему организатору или судье, если у вас есть какие-либо вопросы относительно ваших результатов. \ ~ \ Line \ par \ pard \ nowidctlpar \ sl240 \ slmult1 \ ul \ b \ par \ ulnone \ b0 \ fs24 \ par \ trowd \ trgaph208 \ trleft-158 \ trbrdrl \ brdrs \ brdrw10 \ brdrbtw \ brdrbar \ trbrdrt \ brdrs \ brdrw10 \ trbrdrr \ brdrs \ brdrw10 \ trbrdrb \ brdrs \ brdrw103 \ trpaddlrb \ brdrs \ brdrw103 \ trpaddrw108 \ trpaddlrw103 \ trpaddlr \ clvertalc \ clcfpat9 \ clcbpat9 \ clshdng10000 \ clbrdrl \ brdrw20 \ brdrs \ brdrcf1 \ clbrdrt \ brdrw20 \ brdrs \ brdrcf1 \ clbrdrr \ brdrw20 \ brdrs \ brdrcf1 \ brdrdrs \ brdrdrs \ brdr7220 \ brdrdrs \ brdr7220 \ pard \ intbl \ nowidctlpar \ sl240 \ slmult1 \ cf7 ПОЛИТИКА ОТМЕНА \ ячейка \ строка \ pard \ nowidctlpar \ sl240 \ slmult1 \ cf1 \ ul \ b \ fs20 \ par \ pard \ ulnone \ b0 \ fs22 Применяется следующая политика отмены: \ cf0 \ f2 \ fs24 \ lang9 Для отмены заявок не позднее 17:00 по времени Аляски 28 марта 2021 г. \ f4 \ endash \ f2 Полный возврат средств.\ line При аннулировании заявок после 17:00 28 марта 2021 г. \ f4 \ endash \ f2 возврат средств не производится, если место не может быть заполнено, и в этом случае будет произведено возмещение за вычетом 5 долларов США за обработку. \ par \ pard \ qc \ b \ f1 \ fs22 Все отмены должны быть отправлены по электронной почте [email protected] \ par \ pard \ sl240 \ slmult1 \ cf1 \ b0 \ f0 \ lang1033 \ par \ pard \ nowidctlpar \ sl240 \ slmult1 \ highlight7 \ b \ par \ pard \ sl240 \ slmult1 \ highlight0 \ b0 Стартовые взносы не возвращаются, если собака или хендлер отсутствует, дисквалифицирована или оправдана официальным лицом или судьей NACSW.\ номинал \ ~ \ номинал \ i Настоящая политика отмены и возврата средств применяется к участникам, которые должны отказаться от участия по любой причине, включая, помимо прочего, травму, болезнь, чрезвычайную ситуацию, а также личные или служебные причины. \ i0 \ par \ ~ \ номинал \ i Если ОРТ не может быть проведен или завершен из-за пожара, гражданских беспорядков, стихийных бедствий, чрезвычайного положения или по любой другой причине, не зависящей от организаторов теста, то организатор ОРТ сделает все возможное, чтобы обеспечить частичный возврат денежных средств после возмещения их расходов.\ line \ line \ i0 \ par \ pard \ nowidctlpar \ sl240 \ slmult1 \ cf7 \ fs24 \ par \ trowd \ trgaph208 \ trleft-158 \ trbrdrl \ brdrs \ brdrw10 \ brdrbtw \ brdrbar \ trbrdrt \ brdrs \ brdrw10 \ trbrdrr \ brdrs \ brdrw10 \ trbrdrb \ brdrs \ brdrw103 \ trpaddlrb \ brdrs \ brdrw103 \ trpaddrw108 \ trpaddlrw103 \ trpaddlr \ clvertalc \ clcfpat9 \ clcbpat9 \ clshdng10000 \ clbrdrl \ brdrw20 \ brdrs \ brdrcf1 \ clbrdrt \ brdrw20 \ brdrs \ brdrcf1 \ clbrdrr \ brdrw20 \ brdrs \ brdrcf1 \ brdrdrs \ brdrdrs \ brdr7220 \ brdrdrs \ brdr7220 \ pard \ intbl \ nowidctlpar \ sl240 \ slmult1 ДЕТАЛИ МЕСТОПОЛОЖЕНИЯ \ ячейка \ строка \ pard \ nowidctlpar \ sl240 \ slmult1 \ cf8 \ highlight7 \ b \ fs22 \ par \ cf1 \ highlight0 \ ul Часы работы тестовой площадки \ b0 \ highlight2 \ par \ pard \ qc \ cf0 \ highlight0 \ ulnone \ f1 \ lang9 9:00 AM \ f5 \ endash \ f1 17:00 Подробная информация о времени регистрации и брифинге будет отправлена ​​примерно за 1 неделю до ORT.\ номинал \ pard \ nowidctlpar \ sl240 \ slmult1 \ cf1 \ ul \ f0 \ lang1033 \ par \ b Особые условия сайта \ par \ pard \ cf0 \ ulnone \ b0 \ f1 \ lang9 Подъездная дорожка и парковка \ f5 \ endash \ f1 гравийные, а парковка с уклоном \ f5 \ endash \ f1 также доступна на улице от парковки до здания, по которому вам, возможно, придется перемещаться неровная местность. \ line Испытательная зона может находиться на траектории полета вылетающих рейсов в восточном направлении, если того требуют погодные условия. Обратите внимание на шум реактивной струи, если у вас есть собака, чувствительная к шуму.\ line Испытательная площадка на бетонном полу с резиновым покрытием. \ b \ line \ par \ b0 Из-за непредвиденных обстоятельств, таких как погода или изменение доступности участков в этом месте, ожидаемая тестовая зона, парковка и / или пути могут быть изменены в последнюю минуту. \ par \ pard \ nowidctlpar \ sl240 \ slmult1 \ cf1 \ b \ f0 \ fs24 \ lang1033 \ par \ pard \ qc \ ul Как добраться: \ par \ pard \ line \ cf0 \ f1 \ fs22 \ lang9 Физическое расположение ОРТ: \ ulnone \ b0 В \ ul \ b СКЛАД Кампус \ ulnone \ b0 находится в задней части собственности \ ~ at \ par \ b 511 West 54th Avenue, Анкоридж, Аляска \ b0 \ par \ pard {\ pntext \ f7 \ ‘B7 \ tab} {\ * \ pn \ pnlvlblt \ pnf7 \ pnindent360 {\ pntxtb \’ B7}} \ nowidctlpar \ fi-360 \ li720 \ sb100 \ sa100 \ tx720 \ b \ i Это \ b0 \ ul \ b не \ ulnone наш почтовый адрес \ b0 \ i0 \ par {\ pntext \ f7 \ ‘B7 \ tab} \ b \ i На самом здании нет номера, поэтому внимательно прочтите инструкции ниже \ b0 \ i0 \ par {\ pntext \ f7 \ ‘B7 \ tab} \ b \ i \ ul не \ ulnone останавливаться у дома, расположенного на улице… \ ~ не принадлежит AK Dog Sports! \ ~ \ b0 \ i0 \ par \ pard \ nowidctlpar \ sl240 \ slmult1 Мы находимся на Арктическом бульваре, недалеко от дороги международного аэропорта. Если по направлению \ ul \ b South \ ulnone \ b0, по Арктике пройдите мимо International, а слева — 54-е. Центр оформления мебели Бейли находится на углу. Поверните на 54-ю, пройдите примерно 3 квартала. Слева вы увидите зеленый дом с желтой отделкой и подъездную дорожку, которая немного спускается вниз. \ ~ В почтовом ящике дома \ ~ написано 511 W.54-е. \ ~ НЕ ОСТАНАВЛИВАЙТЕСЬ В ДОМЕ … \ ~ Ищите большое желтовато-коричневое металлическое здание в нижней части наклонной подъездной дорожки со знаком спорта с собаками Аляски на фасаде. \ ~ \ Line \ line Если вы направляетесь в сторону \ ul \ b North \ ulnone \ b0 на Арктике ищите 54-е место справа от дороги перед International. Центр оформления мебели Бейли находится на углу. Поверните на 54-ю, пройдите примерно 3 квартала. Слева вы увидите зеленый дом с желтой отделкой и подъездную дорожку, которая немного спускается под гору. В почтовом ящике дома \ ~ написано 511 W.54-е. \ ~ НЕ ОСТАНАВЛИВАЙТЕСЬ В ДОМЕ … \ ~ Ищите большое желтовато-коричневое металлическое здание в нижней части наклонной подъездной дороги со знаком спорта с собаками Аляски на фасаде. \ Line \ cf1 \ b \ f0 \ fs24 \ lang1033 \ par \ fs22 \ par \ ul Реактивная парковка для собак \ line \ cf0 \ ulnone \ b0 \ f1 \ lang9 В этом тесте будет реактивная парковка для собак для тех, кто хочет припарковаться в этой области, однако парковка узкая и не будет отдельных горшков или разогревайте ящики. \ номинал \ pard Из-за непредвиденных обстоятельств, таких как погода или изменение доступности областей в этом месте, эта политика может быть изменена в последнюю минуту.\ номинал \ pard \ nowidctlpar \ sl240 \ slmult1 \ cf1 \ b \ f0 \ lang1033 \ par \ ul Незнакомые собаки \ par \ pard \ qc \ cf0 \ ulnone \ f1 \ fs24 \ lang9 \ par \ pard \ b0 \ fs22 Мы не можем размещать собак для добровольцев или зрителей. \ номинал Мы не можем разместить в этом месте собак, которые не заходили в дом. \ Par \ pard \ nowidctlpar \ sl240 \ slmult1 \ cf1 \ highlight2 \ f0 \ lang1033 \ par \ highlight0 \ ul \ b Стоянка для автофургонов \ номинал \ highlight4 \ ulnone \ b0 \ par \ cf0 \ highlight0 \ f1 \ lang9 Мы не можем разместить стоянку для автофургонов в этом месте.\ cf1 \ f0 \ lang1033 \ par \ номинал \ номинал \ pard \ sl240 \ slmult1 \ ul \ b Курение \ par \ pard \ cf0 \ ulnone \ b0 \ f1 \ lang9 Курение запрещено на этом тестовом сайте, в том числе использование электронных сигарет \ par Курение разрешено за пределами кампуса на улице вдали от объекта для удобства других. \ номинал \ pard \ nowidctlpar \ sl240 \ slmult1 \ cf1 \ b \ f0 \ lang1033 \ par \ ul Hotels \ par \ pard \ qc \ cf0 \ ulnone \ b0 \ f1 \ lang9 Список местных отелей доступен здесь: {{\ field {\ * \ fldinst {HYPERLINK http: // www.hotels.com}} {\ fldrslt {http: //www.hotels.com \ ul0 \ cf0}}}} \ b \ f1 \ fs24 \ par \ pard \ nowidctlpar \ sl240 \ slmult1 \ cf1 \ highlight2 \ b0 \ f0 \ fs22 \ lang1033 \ line \ highlight0 \ par \ highlight4 \ ul \ fs20 \ par \ trowd \ trgaph208 \ trleft-158 \ trbrdrl \ brdrs \ brdrw10 \ brdrbtw \ brdrbar \ trbrdrt \ brdrs \ brdrw10 \ trbrdrr \ brdrs \ brdrw10 \ trbrdrb \ brdrs \ brdrw103 \ trpaddlrb \ brdrs \ brdrw103 \ trpaddrw108 \ trpaddlrw103 \ trpaddlr \ clvertalc \ clcfpat9 \ clcbpat9 \ clshdng10000 \ clbrdrl \ brdrw20 \ brdrs \ brdrcf1 \ clbrdrt \ brdrw20 \ brdrs \ brdrcf1 \ clbrdrr \ brdrw20 \ brdrs \ brdrcf1 \ brdrdrs \ brdrdrs \ brdr7220 \ brdrdrs \ brdr7220 \ pard \ intbl \ nowidctlpar \ sl240 \ slmult1 \ cf7 \ highlight0 \ ulnone \ fs24 ИНФОРМАЦИЯ О СОБАКЕ \ cell \ row \ pard \ nowidctlpar \ sl240 \ slmult1 \ cf1 \ b \ fs22 \ par \ ul Правила в отношении собак \ b0 \ par \ pard \ nowidctlpar \ sl240 \ slmult1 \ tx90 \ tx432 \ ulnone Все собаки должны быть на поводке 6 \ rquote, когда они не проходят тестирование.ЗАПРЕЩАЕТСЯ использовать гибкие поводки, пока вы не участвуете в тесте. \ Par Реактивные собаки и / или собаки с ограниченными физическими возможностями должны носить красную бандану (которая будет предоставлена ​​проводником собаки), чтобы сигнализировать другим о том, что собаке нужно дополнительное пространство для других собак. \ Par Собаки будут находиться в туалете в специально отведенных местах, и проводники должны собирать и утилизировать отходы. \ Par Собаки и проводники должны оставаться в специально отведенных местах, чтобы не загрязнять пространство для тестирования, не прерывать тест или не просматривать любую часть тестирования.\ номинал \ pard \ nowidctlpar \ sl240 \ slmult1 \ tx90 \ tx450 Участники должны рассчитывать, что они будут держать своих собак в клетке или транспортном средстве, когда не тестируют, и должны спланировать создание собственной тени, используя навесы или теневую ткань. \ номинал Обратитесь к информации о местонахождении (выше) для получения информации о собаках, которые не заходили внутрь. \ номинал \ pard \ nowidctlpar \ sl240 \ slmult1 \ par \ ul \ b Самки в сезоне \ b0 \ par \ ulnone Будет разрешено бегать в штанах \ ldblquote \ rdblquote в самом конце команды после того, как все остальные собаки будут оценены.Сезонные самки будут припаркованы и посажены на горшках отдельно от других собак во время проведения теста, если позволяет материально-техническая база. Хендлеры должны связаться с [email protected], чтобы можно было составить план для собаки. \ Line \ par \ fs24 \ par \ trowd \ trgaph208 \ trleft-158 \ trbrdrl \ brdrs \ brdrw10 \ brdrbtw \ brdrbar \ trbrdrt \ brdrs \ brdrw10 \ trbrdrr \ brdrs \ brdrw10 \ trbrdrb \ brdrs \ brdrw103 \ trpaddlrb \ brdrs \ brdrw103 \ trpaddrw108 \ trpaddlrw103 \ trpaddlr \ clvertalc \ clcfpat9 \ clcbpat9 \ clshdng10000 \ clbrdrl \ brdrw20 \ brdrs \ brdrcf1 \ clbrdrt \ brdrw20 \ brdrs \ brdrcf1 \ clbrdrr \ brdrw20 \ brdrs \ brdrcf1 \ brdrdrs \ brdrdrs \ brdr7220 \ brdrdrs \ brdr7220 \ pard \ intbl \ nowidctlpar \ sl240 \ slmult1 \ cf7 ДРУГАЯ ИНФОРМАЦИЯ \ ячейка \ строка \ pard \ nowidctlpar \ sl240 \ slmult1 \ cf1 \ b \ fs22 \ par \ ul Запросы ADA \ par \ ulnone \ b0 Если вы являетесь инвалидом и хотите запросить жилье (а), отправьте электронное письмо ADA @ nacsw.сеть. Пожалуйста, укажите свое имя, дату ОРТ, имя организатора ОРТ и запрашиваемое вами жилье. \ Par \ line \ par \ pard \ sl240 \ slmult1 \ ul \ b Зрители \ ulnone \ b0 \ line Чтобы свести к минимуму количество людей на площадке, все, кто не участвует в волонтерских программах, не участвует в соревнованиях или не выполняет обязанности, должны оставаться дома, за исключением случаев, когда кому-то \ b требуется поддержка \ b0 для поездка на мероприятие. В настоящее время просмотр запрещен. При обыске должен присутствовать взрослый, сопровождающий несовершеннолетнего. Все участники пробного сайта должны подписать отказ от прав, предоставленный организатором, и соблюдать все требования мероприятия.\ номинал \ номинал \ pard \ nowidctlpar \ sl240 \ slmult1 \ par \ pard \ sl240 \ slmult1 \ ul \ b Ветеринары \ b0 \ par \ pard \ cf0 \ ulnone \ f2 \ fs24 \ lang9 \ line Неотложная помощь домашним животным \ par \ b 2320 E Dowling Rd \ b0 \ line \ b Anchorage, AK 99507 \ b0 \ par Или \ par Больница для животных Midnight Sun и скорая помощь \ par \ pard \ sl240 \ slmult1 \ b 545 East Tudor Road \ line Anchorage, AK 99507 \ cf1 \ highlight2 \ b0 \ f0 \ fs22 \ lang1033 \ line \ highlight0 \ ul \ b \ par \ pard \ nowidctlpar \ sl240 \ slmult1 Rules \ par \ ulnone \ b0 Полные правила доступны по адресу {\ f3 \ fs24 {\ field {\ * \ fldinst {HYPERLINK «http: // www.nacsw.net «}} {\ fldrslt {\ ul \ cf3 \ cf3 \ ul \ f0 \ fs22 www}}}} {\ f3 \ fs24 {\ field {\ * \ fldinst {ГИПЕРССЫЛКА» http: //www.nacsw .net «}} {\ fldrslt {\ ul \ cf3 \ cf3 \ ul \ f0 \ fs22.}}}} {\ cf1 \ ulnone \ f3 \ fs24 {\ field {\ * \ fldinst {ГИПЕРССЫЛКА» http: // www.nacsw.net «}} {\ fldrslt {\ ul \ cf3 \ cf3 \ ul \ f0 \ fs22 NACSW}}}} {\ cf1 \ ulnone \ f3 \ fs24 {\ field {\ * \ fldinst {ГИПЕРССЫЛКА» http : //www.nacsw.net «}} {\ fldrslt {\ ul \ cf3 \ cf3 \ ul \ f0 \ fs22.}}}} {\ cf1 \ ulnone \ f3 \ fs24 {\ field {\ * \ fldinst { ГИПЕРССЫЛКА «http://www.nacsw.net»}} {\ fldrslt {\ ul \ cf3 \ cf3 \ ul \ f0 \ fs22 net}}}} \ cf1 \ ulnone \ f0 \ fs22.\ ul \ b Каждый участник обязан прочитать и понять текущую книгу правил NACSW \ ’99 до участия в мероприятии NACSW. \ par \ ulnone \ b0 \ line \ par \ ul \ b Ответственность \ par \ ulnone \ b0 Регистрируясь для прохождения теста ORT, участник настоящим принимает на себя все риски и ответственность за несчастные случаи и / или ущерб, нанесенный ему / себе, своему имуществу или другим лицам, в результате действий его / его собака. Участник прямо соглашается с тем, что NACSW, Alaska Dog Sports LLC и ее правопреемники или любое другое лицо или лица из указанных групп не несут личной или коллективной ответственности ни при каких обстоятельствах за травмы и / или нанесенный ей ущерб. / самому себе, за утрату или повреждение собственности, будь то из-за неконтролируемых собак или небрежности любого члена указанной группы, или по любой другой причине или причинам.Участник также соглашается принять на себя всю финансовую ответственность, которая может быть понесена NACSW из-за его действий или действий своих собак, независимо от причины. \ Par \ b \ номинал \ ul Sportsmanship \ ulnone \ b0 \ line Ожидается, что все участники / хендлеры и зрители будут следовать правилам NACSW и демонстрировать хорошее спортивное мастерство. Это включает в себя обеспечение того, чтобы места размещения шкуры были неизвестны обработчику до тестирования и держались в секрете от других конкурентов. Пожалуйста, не обсуждайте размещение укрытий или какие-либо детали поведения собаки, которые могут предоставить информацию или подсказки другим хендлерам или могут быть подслушаны.\ номинал \ line \ par \ ul \ b Вопросы \ ulnone \ b0 \ line \ cf0 \ f1 \ lang9 По всем вопросам, связанным с тестом, обращайтесь к Питу или Лизе Саммерс [email protected] \ cf1 \ f0 \ lang1033 \ par \ номинал \ fs16 \ par \ highlight4 \ par \ highlight0 \ fs20 \ par \ номинал \ номинал \ pard \ nowidctlpar \ b \ f1 \ fs24 \ lang9 Alaska Dog Sports llc \ tab \ par NACSW \ ’99 \ f5 \ endash \ f1 Тест на распознавание запаха \ cf0 \ b0 \ f2 \ par \ номинал \ b \ f1 4 апреля 2021 г. \ par \ b0 \ f2 \ par \ b \ f1 Складской кампус ООО «Аляска Дог Спортс» \ линия 511 W.54 \ super th \ nosupersub Ave, Анкоридж, AK \ par \ b0 \ номинал \ pard \ sl240 \ slmult1 \ b 35 долларов США за первый запах и 15 долларов США за каждый дополнительный запах на собаку \ cf1 Регистрационный сбор — Оплата чеком, денежным переводом или PayPal \ line \ f0 \ fs20 \ lang1033 \ par Отправьте эту заполненную форму по почте USPS с указанием регистрационного взноса по адресу: \ par \ f1 \ fs24 \ lang9 Alaska Dog Sports LLC \ line P.O. Box 232521 \ line Anchorage, AK \ line 99523-2521 \ line \ highlight2 \ b0 \ f0 \ fs20 \ lang1033 \ par \ highlight0 \ par \ pard \ sl240 \ slmult1 Вопросы: Контактное лицо: \ f1 \ fs24 \ lang9 Пит Саммерс 907.441-8008 или [email protected] \ f0 \ fs20 \ lang1033 \ par \ pard \ sl240 \ slmult1 \ par Запах: \ tab \ f6 \ u9633? \ F0 Береза ​​\ ~ \ ~ \ ~ \ ~ \ ~ \ ~ \ f6 \ u9633? \ F0 Анис \ ~ \ ~ \ f6 \ u9633? \ F0 Гвоздика \ par \ номинал Дата (даты) тестирования: \ fs24, воскресенье \ f1 \ lang9, 04 апреля 2021 г. \ f0 \ fs20 \ lang1033 \ par \ номинал Имя собаки \ rquote \ ~ _____________________________________________________ \ par \ номинал Порода (и) ______________________ \ ~ \ par \ номинал \ b Собака \ rquote s \ b0 NACSW #________________ \ par \ номинал Имя обработчика \ rquote _____________________________________________________ \ par \ номинал \ b Обработчик \ rquote s \ b0 Членство в NACSW № ____________________________________ \ par \ номинал Адрес _______________________________________________________ \ ~ \ par \ номинал Город __________________ \ ~ Штат _________ Почтовый индекс ________ \ номинал \ номинал Телефон, по которому с вами легко связаться __________________________ \ ~ \ line \ line Адрес электронной почты _____________________________________________________ \ par \ номинал \ b ОРТ должен быть принят и сдан как минимум за 14 дней до даты открытия проб, чтобы иметь право на участие в первом периоде розыгрыша.\ b0 \ номинал \ номинал \ b Пожалуйста, свяжитесь с хозяином как минимум за 1 день до ОРТ, если ваша сука будет в сезон. \ b0 \ par \ номинал Все подтверждения будут отправлены по электронной почте с вложением в течение 7 дней с момента получения полной регистрационной формы и оплаты. \ ~ Если вам требуется подтверждение через USPS, вы должны предоставить конверт с маркой и обратным адресом. \ Par \ номинал Я / мы настоящим принимаем на себя все риски и ответственность за несчастные случаи и / или ущерб, нанесенный мне, моей собственности или другим лицам, в результате действий моей собаки.

Добавить комментарий

Ваш адрес email не будет опубликован. Обязательные поля помечены *